Questions

¡Supera tus tareas y exámenes ahora con Quizwiz!

A man saved the life of his friend's wife who thereafter changed her will to leave the man $1,000. However, upon the wife's death she had no property except an undivided interest in real estate held in tenancy by the entirety of the husband. The property had been purchased by the husband from an inheritance. After the wife died, the husband signed and delivered to the man the following instrument: "In consideration of [the man]'s saving my wife's life and his agreement to bring no claims against my estate based on her will, I hereby promise to pay [the man] $1,000." Upon the husband's death, the man filed a claim for $1,000. The husband's executor contested the claim on the ground that the instrument was not supported by sufficient consideration. In most states, would the man's saving of the wife's life be regarded as sufficient consideration for the husband's promise?

No, because the husband had not asked the man to save her. In order to be enforceable, a promise must be supported by bargained-for consideration. Consideration is bargained for if it is sought by the promisor and given by the promisee in exchange for the promise. Here, the husband did not make the promise to pay $1,000 in exchange for the man's act, nor did the man save the wife's life in exchange for the husband's promise. The man's act would be considered past consideration, which is not sufficient to satisfy the bargain requirement.

A chemist, a citizen of State A, brought suit against a dry cleaner, a citizen of State B, in a state court in State A over a business agreement between them. The dry cleaner had purposefully availed himself through advertisements in State A, but the dry cleaner has never physically been to State A. Which of the following is true?

To know whether the State A state court can have personal jurisdiction over the dry cleaner, it is necessary to analyze State A's long-arm statute A state's long-arm statute cannot give its courts more powers than are allowed under the U.S. Constitution but can give them less. Therefore, it is necessary to consult State A's long-arm statute to determine what constitutes personal jurisdiction in State A.

A man who had become very drunk left a bar and started to walk home. Another patron of the bar, who had observed the man's condition, followed him. The patron saw the man stumble and fall to the ground near an alley. The patron then began to pull out a gun but saw that the man had passed out asleep in the gutter. The patron reached into the man's pocket, grabbed his wallet, and started to walk away. When the patron heard police officers approaching, he dropped the wallet and ran off. The crimes below are listed in descending order of seriousness. What is the most serious crime for which the patron properly could be convicted?

Larceny. Larceny requires a trespassory taking and carrying away of the personal property of another with intent to steal it. All the elements were satisfied here. The asportation (carrying away) element requires movement of only a slight distance, so it was satisfied here even though the patron ultimately discarded the wallet.

The Sports Championship Revenue Enhancement Act is a federal statute that was enacted as part of a comprehensive program to eliminate the federal budget deficit. That act imposed, for a period of five years, a 50% excise tax on the price of tickets to championship sporting events. Such events included the World Series, the Super Bowl, major college bowl games, and similar championship sports events. This federal tax is probably

constitutional, because an act of Congress that appears to be a revenue raising measure on its face is not rendered invalid because it may have adverse economic consequences for the activity taxed. Even though the tax is high at 50%, Congress has broad power to raise revenue through taxes. A measure that appears to be revenue raising on its face is not invalid merely because it may have adverse economic consequences for the activity being taxed.

A farmer contracted to sell 100,000 bushels of wheat to a buyer. When the wheat arrived at the destination, the buyer discovered that the farmer had delivered only 96,000 bushels. The buyer sued the farmer for breach of contract. At the trial of the case, the court found that the written contract was intended as a complete and exclusive statement of the terms of the agreement. The farmer offered to prove that in the wheat business, a promise to deliver a specified quantity is considered to be satisfied if the delivered quantity is within 5% of the specified quantity. The buyer objected to the offered evidence. Is the court likely to admit the evidence offered by the farmer?

Yes, because the offered evidence explains or supplements the agreement by usage of trade This transaction involves a sale of goods and is subject to UCC Article 2. Under Article 2, evidence of trade usage that can be construed as reasonably consistent with an agreement's express language is admissible to interpret or supplement an agreement. The majority rule provides that trade usage will be viewed as consistent with an agreement's express language unless the usage completely negates specific express language. The trade usage allowing for a variation of up to 5% does not completely negate but rather qualifies the express language calling for the delivery of 100,000 bushels of wheat.

An appropriations act passed by Congress over the President's veto directs that one billion dollars "shall be spent" by the federal government for the development of a new military weapons system, which is available only from the Arms Corporation. On the order of the President, the Secretary of Defense refuses to authorize a contract for purchase of the weapons system. The Arms Corporation sues the Secretary of Defense alleging an unlawful withholding of these federal funds. The strongest constitutional argument for the Arms Corporation is that

Congress' power to appropriate funds includes the power to require that the funds will be spent as directed. While not an independent source of power, the Necessary and Proper Clause does give Congress the power to make all laws necessary and proper for carrying out any power granted to any branch of the federal government. Thus, the strongest argument that the Arms Corporation has is that Congress' power to appropriate funds to the executive branch also includes the power to direct how the funds are spent.

A police officer had a hunch, not amounting to probable cause or reasonable suspicion, that a man was a drug dealer. One day while the officer was on highway patrol, her radar gun clocked the man's car at 68 mph in an area where the maximum posted speed limit was 65 mph. The officer's usual practice was not to stop a car unless it was going at least 5 mph over the posted limit, but contrary to her usual practice, she decided to stop the man's car in the hope that she might discover evidence of drug dealing. After she stopped the car and announced that she would be writing a speeding ticket, the officer ordered the man and his passenger to step out of the car. When the passenger stepped out, the officer saw that the passenger had been sitting on a clear bag of what the officer immediately recognized as marijuana. The officer arrested both the man and the passenger for possession of marijuana. At their joint trial, the man and the passenger claim that their Fourth Amendment rights were violated because the officer improperly (1) stopped the car for speeding as a pretext for investigating a hunch rather than for the stated purpose of issuing a traffic ticket and (2) ordered the passenger to step out of the car even though there was no reason to believe that the passenger was a criminal or dangerous. Are the man and the passenger correct?

No, as to both the stop of the car and the officer's order that the passenger step out of the car The stop of the car was constitutional, because it was objectively justifiable (regardless of the officer's subjective motivation), and both the driver and any passengers may be ordered to step out of a car during a lawful traffic stop.

A mother rushed her eight-year-old daughter to the emergency room at a local hospital after the child fell off her bicycle and hit her head on a sharp rock. The wound caused by the fall was extensive and bloody. The mother was permitted to remain in the treatment room, and held the child's hand while the emergency room physician cleaned and sutured the wound. During the procedure, the mother said that she was feeling faint and stood up to leave the room. While leaving the room, the mother fainted and, in falling, struck her head on a metal fixture that protruded from the emergency room wall. She sustained a serious injury as a consequence. If the mother sues the hospital to recover damages for her injury, will she prevail?

No, because there is no evidence that the hospital's personnel failed to take reasonable steps to anticipate and prevent the mother's injury An invitee is owed the duty of reasonable inspection to find hidden dangers. The mother is an invitee because she entered the premises under the hospital's implied invitation. The hospital has a duty to keep the premises in a reasonably safe condition for her use. The hospital is not liable because it did not fail to use reasonable care in connection with the fixture. Here, there is no evidence that the hospital's personnel failed to take reasonable steps to anticipate and prevent the mother's injury. Therefore, the hospital will not be liable.

An eight-year-old child went to the grocery store with her mother. The child pushed the grocery cart while her mother put items into it. The child's mother remained near the child at all times. Another customer in the store noticed the child pushing the cart in a manner that caused the customer no concern. A short time later, the cart the child was pushing struck the customer in the knee, inflicting serious injury. If the customer brings an action, based on negligence, against the child, the child's best argument in defense would be that

The child exercised care commensurate with her age, intelligence, and experience. This choice gives a child-appropriate negligence standard of care. The customer's claim for negligence will be allowed, but the child will only be held to the standard of care expected of "a reasonable child" of the same age, training, maturity, experience, and intelligence.

A state statute requires the permanent removal from parental custody of any child who has suffered "child abuse." That term is defined to include "corporal punishment of any sort." A father very gently spanks his six-year-old son on the buttocks whenever he believes that spanking is necessary to enforce discipline on him. Such a spanking occurs not more than once a month and has never physically harmed the child. The state files suit under the statute to terminate the father's parental rights solely because of these spankings. The father defends only on the ground that the statute in question is unconstitutional as applied to his admitted conduct. In light of the nature of the rights involved, which of the following is the most probable burden of persuasion on this constitutional use?

The state has the burden of persuading the court that the application of this statute to the father is necessary to accomplish a compelling state interest Parents have a fundamental right to make a decision concerning the care and control of their children. Therefore, restrictions on their ability to make those decisions are subject to strict scrutiny. Under strict scrutiny, the government has the burden of demonstrating that the restriction is necessary to accomplish a compelling state interest. Therefore, the state must prove that preventing the father from spanking his child is necessary to accomplish a compelling state interest.

A husband and wife acquired land as common law joint tenants with right of survivorship. One year later, without his wife's knowledge, the husband executed a will devising the land to his best friend. The husband subsequently died. Is the wife now the sole owner of the land?

Yes, because the devise to the friend did not sever the joint tenancy Although as a general rule a joint tenant's interest is freely alienable during his or her lifetime without the consent of the other joint tenant, that interest cannot be devised in a will. In this case, on the death of the husband, the wife's interest in the joint tenancy immediately swelled and she became the sole owner of the land as the surviving joint tenant.

A man was short of money. He decided to go into his neighbor's house to take the neighbor's silverware and then to sell it. That night, while the neighbor was away, the man entered by picking the lock on the front door. He picked up a chest of silverware from the dining room and went out the front door of the house to his car. As he was putting the chest of silverware into the trunk, he had second thoughts and decided that he did not wish to become a thief. He reentered the house and replaced the chest of silverware where he had found it. As he came out of the house the second time, he was arrested by the police, who had been called by a passerby. The man is

guilty of burglary and larceny The man took the neighbor's silverware, and carried it away with the intent to permanently deprive the owner of his possession, so the man is guilty of larceny. The man broke and entered the neighbor's dwelling at night with the intent to commit the larceny, so he is also guilty of burglary. The fact that the man changed his mind about the crimes after he had completed them does not negate either charge.

While browsing in a clothing store, a defendant decided to take a purse without paying for it. She placed the purse under her coat and took a couple of steps toward the exit. She then realized that a sensor tag on the purse would set off an alarm. She placed the purse near the counter from which she had removed it. The defendant has committed

larceny, because she took the purse from its original location and concealed it with the intent to steal Larceny is the taking possession and carrying away of the personal property of another, without the owner's consent and with the intent to permanently deprive the owner of the property. The defendant took the purse and concealed it under her coat with the intent to permanently deprive the owner of the purse. Therefore, the defendant committed larceny.

Statutes in a jurisdiction define criminal assault as "an attempt to commit a criminal battery" and criminal battery as "causing an offensive touching." As a man was walking down the street, a gust of wind blew his hat off. The man reached out, trying to grab his hat, and narrowly missed striking a woman in the face with his hand. The woman, fearful of being struck by the man, pushed the man away. If charged with criminal assault, the man should be found

not guilty, because he did not intend to hit the woman Under the statute given, assault is the attempt to commit a criminal battery. To attempt to commit a crime, one must have the intent that the crime be committed and take a substantial step towards its commission. The man had no intent to commit a criminal battery; he was attempting to grab his hat.

A city ordinance makes it unlawful to park a motor vehicle on a city street within ten feet of a fire hydrant. At 1:55 p.m. a man, realizing he must be in the bank before it closed at 2:00 p.m., and finding no other space available, parked his automobile in front of a fire hydrant on a city street. The man then hurried into the bank, leaving his aged neighbor as a passenger in the rear seat of the car. About 5 minutes later, and while the man was still in the bank, a driver was driving down the street. The driver swerved to avoid what he mistakenly thought was a hole in the street and sideswiped the man's car. The man's car was turned over on top of the hydrant, breaking the hydrant and causing a small flood of water. The man's car was severely damaged and the neighbor was badly injured. There is no applicable guest statute. If the neighbor asserts a claim against the man, the most likely result is that the neighbor will

not recover, because a reasonably prudent person could not foresee injury to the neighbor as a result of the man's action It addresses the issue of proximate or legal cause of the injury. The man had a duty of reasonable care to his aged neighbor, but when he negligently parked in a no parking zone, it was not reasonably foreseeable that the type of injury that would result from parking near the hydrant would be a rollover and flood injury. Consequently, the man's liability is cut short, and the neighbor will not prevail against the man for injuries suffered in the rollover.

A landowner owned Blackacre in fee simple, as the land records showed, when he contracted to sell Blackacre to a buyer. Two weeks later, the buyer paid the agreed price and received a warranty deed. A week thereafter, when neither contract nor deed had been recorded and while the owner remained in possession of Blackacre, a creditor properly filed a money judgment against the owner. She knew nothing of the buyer's interest. A statute in the jurisdiction provides: "Any judgment properly filed shall, for ten years from filing, be a lien on the real property then owned or subsequently acquired by any person against whom the judgment is rendered." The recording act of the jurisdiction provides: "No conveyance or mortgage of real property shall be good against subsequent purchasers for value and without notice unless the same be recorded according to law." The creditor brought an appropriate action to enforce her lien against Blackacre in the buyer's hands. If the court decides for the buyer, it will most probably be because

the jurisdiction's recording act does not protect creditors The recording statute identified in the facts is a notice act. To prevail under a notice act, a party must be a bona fide purchaser ("BFP") who recorded her deed without notice of earlier purchasers. To be a BFP, a party must give value for her interest in the land. Although the facts indicate that the creditor filed her judgment without actual notice or record notice of the conveyance (it had not been recorded yet), the recording act in this jurisdiction protects purchasers for value, meaning BFP's. Therefore, if the buyer wins, it is most likely because the creditor was not a BFP.

A federal statute with inseverable provisions established a new five-member national board with broad regulatory powers over the operation of the securities, banking, and commodities industries, including the power to issue rules with the force of law. The statute provides for three of the board members to be appointed by the President with the advice and consent of the Senate. They serve seven-year terms and are removable only for good cause. The other two members of the board were designated in the statute to be the respective general counsel of the Senate and House of Representatives Committees on Government Operations. The statute stipulated that they were to serve on the board for as long as they continued in those positions. Following all required administrative procedures, the board issued an elaborate set of rules regulating the operations of all banks, securities dealers, and commodities brokers. A company that was subject to the board's rules sought a declaratory judgment that the rules were invalid because the statute establishing the board was unconstitutional. In this case, the court should rule that the statute establishing the national board is

unconstitutional, because all members of federal boards exercising executive powers must be appointed by the President or in a manner otherwise consistent with the Appointments Clause of Article II The statute is probably unconstitutional because the members of federal boards must be appointed in a manner consistent with the Appointments Clause of Article II. They must be appointed in this manner because they are members of the executive branch.

Congress passed a bill prohibiting the President from granting a pardon to any person who had not served at least one-third of the sentence imposed by the court which convicted that person. The President vetoed the bill, claiming that it was unconstitutional. Nevertheless, Congress passed it over his veto by a two- thirds vote of each house. This act of Congress is

unconstitutional, because it interferes with the plenary power of the President to grant pardons Although Congress can usually enact laws that the President has vetoed by a two-thirds majority vote (as choice A would suggest), Congress cannot take power away from the President that the Constitution grants. Article II, § 2, cl. 1 states that the President shall have the power to grant pardons except in cases of impeachment. That power is not qualified in any way, and for Congress to attempt to qualify the power is unconstitutional.

On a wholly random basis, a state agency has given a few probationary employees who were not rehired at the end of their probationary period a statement of reasons and an opportunity for a hearing; but the agency has very rarely done so. No statute or rule of the agency required such a statement of reasons or a hearing. The employment of a probationary employee was terminated without a statement of reasons or an opportunity for a hearing. The agency did not even consider whether it should give him either. A suit by the employee requesting a statement of reasons and a hearing will probably be

unsuccessful, because the employee does not have the right to be rehired that is protected by procedural due process There is no inherent right to public employment, but if the job is vested, a property interest will be recognized. Then the employee can only be terminated for cause. Probationary employees do not have any kind of statutory right to be rehired, nor do they have any statutory right to procedures surrounding the decision not to be rehired under this scenario.

Congress provides by statute that any state that fails to prohibit automobile speeds of over 55 miles per hour on highways within the state shall be denied all federal highway construction funding. One of the richest and most highway-oriented states in the country refuses to enact such a statute. The strongest argument that can be made in support of the constitutionality of this federal statute is that

Congress could reasonably believe that the 55 mile-an-hour speed limit will assure that the federal money spent on highways results in greater benefit than harm to the public. Congress has plenary spending power, and may impose reasonable conditions upon acceptance of federal funds. There is a nexus between safety, the speed limit, and the use of federal funds on the highways. Therefore the speeding limit is constitutional.

A landlord owns and operates a 12-story apartment building containing 72 apartments, 70 of which are rented. A pedestrian has brought an action against the landlord alleging that while he was walking along a public sidewalk adjacent to the landlord's apartment building a flower pot fell from above and struck him on the shoulder, causing extensive injuries. The action was to recover damages for those injuries. If the pedestrian proves the foregoing facts and offers no other evidence explaining the accident, will his claim survive a motion for directed verdict offered by the defense?

No, because there is no basis for a reasonable inference that the landlord was negligent. Any time negligence must be inferred, there is a res ipsa loquitur issue. The doctrine of res ipsa loquitur will establish a prima facie case for negligence where (1) the defendant had exclusive control of the instrumentality during the relevant time, and (2) the plaintiff shows that he was not responsible for the injury. For a claim based on res ipsa loquitur to prevail, the pedestrian must show that the landlord had exclusive control of the flowerpot before it fell. In this case, the landlord did not have exclusive control of the flowerpot during the relevant time frame because 70 of the units had tenants, so res ipsa loquitur may not be used to establish negligence. Therefore, the pedestrian will not be able to make his prima facie case, and the landlord's motion for a directed verdict should be granted.

A police officer stopped the defendant for speeding late one night. Noting that the defendant was nervous, he ordered him from the car and placed him under arrest for speeding. By state law, the officer was empowered to arrest the defendant and take him to the nearest police station for booking. He searched the defendant's person and discovered a package of heroin in his jacket pocket. The defendant is charged with possession of heroin. At trial, the defendant's motion to prevent introduction of the heroin into evidence, on the ground that the search violated his federal constitutional rights, will most probably be

denied, because the search was incident to a valid custodial arrest. Authorities may conduct a warrantless search incident to a lawful arrest as long as the scope of the search is constrained to the defendant's wingspan and performed contemporaneously with the arrest. Here, the officer was empowered by state law to arrest the defendant for speeding, the search was within the defendant's wingspan, and the search was contemporaneous with a lawful arrest. Therefore, the search was valid and the defendant's motion should be denied.

The defendant and his friend were fooling around with a pistol in the friend's den. The defendant aimed the pistol in the friend's direction and fired three shots slightly to the friend's right. One shot ricocheted off the wall and struck his friend in the back, killing him instantly. The most serious crime of which the defendant can be convicted is

murder. At common law, murder is defined as the unlawful killing of another human with malice aforethought. By firing multiple shots from a pistol extremely close to his friend, the defendant was acting with reckless indifference to an unjustifiably high risk to human life. This reckless indifference would qualify as malice aforethought, and would support a murder conviction. Because the defendant's actions could support a murder conviction. B, C, and D are incorrect. These are less serious offenses.

A state enacts the Young Adult Marriage Counseling Act, which provides that, before any persons less than 30 years of age may be issued a marriage license, they must receive at least five hours of marriage counseling from a state-licensed social worker. This counseling is designed to assure that applicants for marriage licenses know their legal rights and duties in relation to marriage and parenthood, understand the "true nature" of the marriage relationship, and understand the procedures for obtaining divorces. In a case in which the constitutionality of the Young Adult Marriage Counseling Act is in issue, the burden of persuasion will probably be on the

state, because there is a substantial impact on the right to marry, and that right is fundamental. The right to marry is fundamental, and this statute substantially impacts that right, so the statute would be subject to strict scrutiny. The state bears the burden to show that the statute will pass strict scrutiny.

A state requires licenses of persons "who are engaged in the trade of barbering." It will grant such licenses only to those who are graduates of barber schools located in the state, have resided in the state for two years, and are citizens of the United States. The requirement that candidates for licenses must be citizens is

unconstitutional as a denial of equal protection. Under the Equal Protection Clause, a state statute which uses alienage as a classification must pass strict scrutiny. Here, the classification fails strict scrutiny because the state cannot prove that limiting barber licenses to citizens is necessary to achieve a compelling state interest.

A state statute made it a misdemeanor to construct any building of more than five stories without an automatic fire sprinkler system. A local construction company built in the state a ten-story federal office building. It constructed the building according to the precise specifications of a federal contract authorized by federal statutes. Because the building was built without the automatic fire sprinkler system required by state law, the state prosecuted the private contractor. Which of the following is the company's strongest defense to that prosecution?

As applied, the state sprinkler requirement violates the Supremacy Clause. The contractor built the building to precise specifications of a federal contract authorized by federal statutes. If the state sprinkler requirement conflicts with the federal statutes, the state sprinkler requirement cannot be enforced under the Supremacy Clause. Therefore, this is the strongest defense.

A state has enacted a new election code designed to increase voter responsibility in the exercise of the franchise and to enlarge citizen participation in the electoral process. None of its provisions conflicts with federal statutes. Which of the following is the strongest reason for finding unconstitutional a requirement in the state code that each voter must be literate in English?

The requirement violates the Equal Protection Clause of the Fourteenth Amendment. Literacy requirements have been used in the past to disenfranchise minority voters as a class. English proficiency is not a perfect proxy for being politically informed, and it is more likely that political minorities will be less proficient in English. Under Equal Protection, these types of restrictions would be unconstitutional.

A landowner and a contractor entered into a written contract under which the contractor agreed to build a building and pave an adjacent sidewalk for the landowner at a price of $200,000. Later, while construction was proceeding, the landowner and the contractor entered into an oral modification under which the contractor was not obligated to pave the sidewalk but still would be entitled to $200,000 upon completion. The contractor completed the building. The landowner, after discussions with his landscaper, demanded that the contractor pave the adjacent sidewalk. The contractor refused. Has the contractor breached the contract?

YYes, because there was no consideration for the discharge of the contractor's duty to pave the sidewalk Because it is a contract for services, this contract is governed by the common law of contracts, and not the Uniform Commercial Code. The prevailing common law view is that a modification to a contract requires consideration to be valid. Here, there was no consideration for the elimination of the contractor's duty to pave the sidewalk. The contractor actually promised to do less than he was under a preexisting duty to do, and so there was no enforceable modification of the contract.

A plaintiff suffered from a serious, though not immediately life-threatening impairment of his circulatory system. The plaintiff's cardiologist recommended a cardiac bypass operation and referred the plaintiff to a surgeon. The surgeon did not inform the plaintiff of the 2% risk of death associated with this operation. The surgeon defended his decision not to mention the risk statistics to the plaintiff because the plaintiff "was a worrier and it would significantly lessen his chance of survival to be worried about the nonsurvival rate." The surgeon successfully performed the bypass operation and the plaintiff made a good recovery. However, when the plaintiff learned of the 2% risk of death associated with the operation, he was furious that the surgeon had failed to disclose this information to him, saying that he would have refused the operation if he had known of the risk. If the plaintiff asserts a claim against the surgeon based on negligence, will the plaintiff prevail?

No, because the operation was successful and the plaintiff suffered no harm. Negligence (including medical malpractice) requires proof of duty, breach, causation, and damages. It is not a dignitary tort. If the plaintiff did not suffer an actual injury, he cannot prevail in an action for negligence against his doctor.

Suffering from painful and terminal cancer, a wife persuaded her husband to kill her to end her misery. As they reminisced about their life together and reaffirmed their love for each other, the husband tried to discourage the wife from giving up. The wife insisted, however, and finally her husband held a gun to her head and killed her. The most serious degree of criminal homicide of which the husband can be legally convicted is

murder. The husband, with deliberation and premeditation, and with the intent to kill his wife, put a gun to her head and shot her. The husband is guilty of murder. The motive (which is not the same thing as intent) behind the husband's actions does not excuse this murder and does not negate his premeditation, deliberation, and malice aforethought.

A state legislature recently enacted a statute forbidding public utilities regulated by the state's public service commission to increase their rates more than once every two years. A power company, a public utility regulated by that commission, has just obtained approval of the commission for a general rate increase. The power company has routinely filed for a rate increase every ten to 14 months during the last 20 years. Because of uncertainties about future fuel prices, the power company cannot ascertain with any certainty the date when it will need a further rate increase; but it thinks it may need such an increase sometime within the next 18 months. The power company files an action in the federal district court in the state requesting a declaratory judgment that this new state statute forbidding public utility rate increases more often than once every two years is unconstitutional. Assume no federal statute is relevant. In this case, the court should

dismiss the complaint, because this action is not ripe for decision. There is no injury yet to the power company. It has not yet been denied a rate increase, and it does not even know whether it will need to seek a rate increase before the statute will allow it to. Therefore, there is not yet an actual or imminent injury, and the claim is not yet ripe for decision.

While driving at a speed in excess of the statutory limit, the defendant negligently collided with another car, and the disabled vehicles blocked two of the highway's three northbound lanes. When the plaintiff approached the scene two minutes later, he slowed his car to see if he could help those involved in the collision. As he slowed, he was rear-ended by the driver of another vehicle. The plaintiff, who sustained damage to his car and was seriously injured, brought an action against the defendant to recover damages. The jurisdiction adheres to the traditional common law rules pertaining to contributory negligence. If the defendant moves to dismiss the action for failure to state a claim upon which relief may be granted, should the motion be granted?

No, because a jury could find that the plaintiff's injury arose from a risk that was a continuing consequence of the defendant's negligence This question addresses the issue of causation in a negligence action, and causation requires that the defendant's negligence be both the actual and legal cause of the plaintiff's injuries. The defendant is the actual cause of the plaintiff's injuries because but for the defendant's negligent speeding, the plaintiff would not have been injured. Furthermore, it is possible the defendant is the legal cause of the plaintiff's injuries because it is foreseeable that speeding would cause an accident, and that the accident would create a dangerous road condition leading to subsequent accidents.

Insurance is provided in a particular state only by private companies. Although the state insurance commissioner inspects insurance companies for solvency, the state does not regulate their rates or policies. An insurance company charges higher rates for burglary insurance to residents of one part of a county in the state than to residents of another section of the same county because of the different crime rates in those areas. The plaintiff is a resident of the county who was charged the higher rate by the insurance company because of the location of her residence. The plaintiff sues the insurance company, alleging that the differential in insurance rates unconstitutionally denies her the equal protection of the law. Will the plaintiff's suit succeed?

No, because the constitutional guarantee of equal protection of the law is not applicable to the actions of these insurance companies The Constitution provides for equal protection of the law, which means that it protects individuals from actions by the state (the Fourteenth Amendment) or the federal government (the Fifth Amendment). Equal protection only restricts private action in extremely specific circumstances, none of which are present in this fact pattern.

A man obtained a bank loan secured by a mortgage on an office building that he owned. After several years, the man conveyed the office building to a woman, who took title subject to the mortgage. The deed to the woman was not recorded. The woman took immediate possession of the building and made the mortgage payments for several years. Subsequently, the woman stopped making payments on the mortgage loan, and the bank eventually commenced foreclosure proceedings in which the man and the woman were both named parties. At the foreclosure sale, a third party purchased the building for less than the outstanding balance on the mortgage loan. The bank then sought to collect the deficiency from the woman. Is the bank entitled to collect the deficiency from the woman?

No, because the woman is not personally liable on the loan The woman took title to the office building subject to the mortgage but did not assume the mortgage debt. The debt is to be satisfied out of the building. The building is the principal, and the man, as transferor, is the only party liable for any deficiency. This situation can be contrasted with one in which a buyer expressly assumes the mortgage debt. In that case, the buyer would be primarily liable for any deficiency and the seller, absent a release by the mortgagee, would be secondarily liable.

A seller conveyed residential land to a buyer by a warranty deed that contained no exceptions and recited that the full consideration had been paid. To finance the purchase, the buyer borrowed 80% of the necessary funds from a bank. The seller agreed to finance 15% of the purchase price, and the buyer agreed to provide cash for the remaining 5%. At the closing, the buyer signed a promissory note to the seller for 15% of the purchase price but did not execute a mortgage. The bank knew of the loan made by the seller and of the promissory note executed by the buyer to the seller. The buyer also signed a note to the bank, secured by a mortgage, for the 80% advanced by the bank. The buyer has now defaulted on both loans. There are no applicable statutes. Which loan has priority?

The bank's loan, because it was secured by a purchase-money mortgage The bank has a purchase-money mortgage, because the loan proceeds were used to help purchase the land. A purchase-money mortgage, executed at the same time as the deed to the land, takes precedence over any other lien that attaches to the property. The seller's loan could also have been secured by a purchase-money mortgage, but it was not; the buyer signed an unsecured note to the seller. The seller also may have had an equitable vendor's lien for the unpaid purchase price, but the deed recites that the full consideration was paid. Therefore, the bank's purchase-money mortgage takes priority over the seller's unsecured loan and any implied equitable vendor's lien even if the bank knew of the vendor's lien.

A defendant wanted to make some money, so she decided to sell cocaine. She asked her neighbor, who was reputed to have access to illegal drugs, to supply her with cocaine so she could resell it. The neighbor agreed and sold the defendant a bag of white powder. The defendant then repackaged the white powder into smaller containers and sold one to an undercover police officer who promptly arrested the defendant. The defendant immediately confessed and said that her neighbor was her supplier. Upon examination, the white powder was found not to be cocaine or any type of illegal substance. If the neighbor knew the white powder was not cocaine but the defendant believed it was, which of the following is correct?

The neighbor is not guilty of attempting to sell cocaine, but the defendant is To attempt to commit a crime, one must have the intent that the crime be committed and take a substantial step towards its commission. The neighbor, knowing that the white powder was not cocaine, did not have the intent to sell cocaine; he intended to sell a look-alike substance. The defendant, on the other hand, had the intent to sell cocaine, and took substantial steps toward the commission of that offense. The fact that it was factually impossible for her to commit the crime of selling cocaine is not a valid defense to the attempt charge. The neighbor is not guilty of attempting to sell cocaine, but the defendant is guilty.

The owner of a house told his neighbor that he was going away for two weeks and asked the neighbor to keep an eye on his house. The neighbor agreed. The owner gave the neighbor a key to use to check on the house. The neighbor decided to have a party in the owner's house. He invited a number of friends. One friend, a pickpocket, went into the owner's bedroom, took some of the owner's rings, and put them in his pocket. Which of the following is true?

The pickpocket is guilty of larceny and the neighbor is not guilty of any crime. The neighbor is not criminally responsible for the pickpocket's theft, and the neighbor had permission to be in the residence. The neighbor is not guilty of any crime. The pickpocket, however, took the owner's property, without the owner's consent, put it in his pocket, and is thus guilty of larceny.

The defendant was arrested and taken to police headquarters, where she was given Miranda warnings. The defendant indicated that she wished to telephone her lawyer and was told that she could do so after her fingerprints had been taken. While being fingerprinted, however, the defendant blurted out, "Paying a lawyer is a waste of money because I know you have me." At trial, the defendant's motion to prevent the introduction of the statement she made while being fingerprinted will most probably be

denied, because the statements were volunteered and not the result of interrogation The exclusionary rule only extends to a defendant's statements made in the course of a custodial interrogation and does not extend to voluntary statements made by the defendant that are not the result of police interrogation or misconduct. Here, the defendant's statement during fingerprinting was voluntarily made and not the result of a custodial interrogation. Therefore, her motion to exclude them should be denied.

A motorist arranged to borrow his friend's car to drive for one day while the motorist's car was being repaired. The friend knew that the brakes on his car were faulty and might fail in an emergency. The friend forgot to tell the motorist about the brakes when the motorist picked up the car, but the friend did telephone the motorist's wife and told her about them. The wife, however, forgot to tell the motorist. The motorist was driving the friend's car at a reasonable rate of speed and within the posted speed limit with the motorist's wife as a passenger. Another car, driven by a woman, crossed in front of the motorist at an intersection and in violation of the traffic signal. The motorist tried to stop, but the brakes failed, and the two cars collided. If the brakes had been in proper working order, the motorist could have stopped in time to avoid the collision. The motorist and his wife were injured. If the motorist asserts a claim against the woman, the motorist will

recover the full amount of his damages, because the motorist himself was not at fault This answer states the reason why the motorist will recover full damages from the woman. The motorist was driving reasonably and attempted to stop when the woman cut him off in violation of a traffic signal. The motorist's inability to stop despite his attempt does not give rise to fault. The woman had a duty to obey the traffic lights and it was foreseeable that breaching her duty and crossing in violation of a light would result in harm to an opposing motorist.

In a writing signed by both parties, a renowned architect agreed for a fee of $25,000 to design and supervise construction of a new house for a famous sculptor, the fee to be paid upon completion of the house. The architect and sculptor got along poorly, and, when the design plans were about two-thirds complete, they had a heated argument over the proper location of a marble staircase. Hoping to avoid such encounters, the architect, without the sculptor's knowledge, assigned to a newly-licensed architect practicing solo, "all of my rights and duties under my design and construction supervision contract with the sculptor." The newly-licensed architect expressly promised the renowned architect to carry out the work to the best of her ability. Assume that the sculptor allowed the newly-licensed architect to proceed with the design work but that the newly-licensed architect, without legal excuse, abandoned the project shortly after construction began. Which of the following legal conclusions are correct?

Both the renewed architect and the newly-licensed architect are liable to the sculptor for any legal damages caused by the newly-licensed architect's default The renowned architect is liable to the sculptor for any legal damages caused by the newly-licensed architect's default because when a party delegates duties under a contract with the consent of the obligee (the sculptor), it does not result in a complete substitution of the delegatee (the newly-licensed architect) for the delegator (the renowned architect). Generally, when a party delegates its duties the delegator will remain liable under the contract even if the delegatee expressly assumes the duties. However, as between the delegator and the delegatee, the delegation places the primary responsibility to perform on the delegatee. The delegator becomes secondarily liable for performance of the duty. Here, the renowned architect cannot unilaterally release himself from his obligation to the sculptor and remains liable for the newly-licensed architect's failure to perform or defective performance. The newly-licensed architect is liable to the sculptor for any legal damages caused by her default because a delegatee (the newly-licensed architect) becomes liable to the obligee (the sculptor) if the delegatee makes a promise to either the obligee or the delegator (the renowned architect) to perform under the contract, and the newly-licensed architect "expressly promised the architect" to perform. However, the sculptor is not indebted to the newly-licensed architect for a prorated portion of the agreed $25,000 architect's fee promised to the renowned architect. This is because the parties did not intend for the contract to be one in which the mutual performances promised by the parties could be split into a number of smaller independent and self-contained sets of matching performances. To the extent that design and construction could be separated, the parties did not manifest this intent. B, C, and D are incorrect.

The plaintiff took a diamond ring to a pawnshop and borrowed $20 on it. It was agreed that the loan was to be repaid within 60 days and if it was not, the pawnshop owner, the defendant, could sell the ring. A week before expiration of the 60 days, the defendant had an opportunity to sell the ring to a customer for $125. He did so, thinking it unlikely that the plaintiff would repay the loan and if he did, the defendant would be able to handle him somehow, even by paying him for the ring if necessary. Two days later, the plaintiff came in with the money to reclaim his ring. The defendant told him that it had been stolen when his shop was burglarized one night and that therefore he was not responsible for its loss: Larceny, embezzlement, and false pretenses are separate crimes in the jurisdiction. It is most likely that the defendant has committed which of the following crimes?

Embezzlement. At common law, larceny is the taking possession and carrying away of the personal property of another, without the owner's consent and with the intent to permanently deprive the owner of said property. At common law, larceny by trick is the obtaining of possession, with the owner's consent, of the property of another by fraud or misrepresentation, with the intent to permanently deprive the owner of said property. At common law, false pretenses is obtaining the possession and title of property of another through fraud or misrepresentation, with the intent to permanently deprive the owner of the property. At common law, embezzlement is the fraudulent conversion of another person's property by someone who had lawful possession of said property. In the question, the defendant had lawful possession of the ring and fraudulently converted it by selling it to another customer for $125. The defendant had a fiduciary relationship with the plaintiff and was properly in possession of the ring, but illegally converted it.

A state requires licenses of persons "who are engaged in the trade of barbering." It will grant such licenses only to those who are graduates of barber schools located in the state, have resided in the state for two years, and are citizens of the United States. Which of the following is the strongest ground on which to challenge the requirement that candidates for barber licenses must have been residents of the state for at least two years?

The Privileges or Immunities Clause of the Fourteenth Amendment Under the Privileges or Immunities Clause, citizens have a fundamental right to travel. This has been interpreted to mean that state durational residency requirements must meet strict scrutiny. See Saenz v. Roe, 526 U.S. 489 (1999). By limiting licenses to candidates who have been residents of the state for at least two years, the state is interfering with the right to travel, and the restriction will be subject to strict scrutiny.

In state court, a ballerina brought suit against a non-diverse manufacturer of an over-the-counter drug that the ballerina ingested as directed on the packaging. The ballerina claims that, as a result of the drug, she suffered serious vision impairment. The ballerina asserted state law product liability claims and claims for improper labeling of the drug in violation of state law requirements. In its defense, the manufacturer asserts that the ballerina's improper labeling claim is preempted by federal law, and that the manufacturer complied with the federal law requirements regarding the labeling of the drug. The manufacturer also filed a timely notice of removal with the local federal district court. The ballerina then moved to remand the case back to state court and asserts that the federal court lacks subject-matter jurisdiction. Should the court grant the ballerina's motion to remand the case back to state court?

The court should grant the ballerina's motion to remand because the court lacks subject-matter jurisdiction over the case The court lacks subject-matter jurisdiction over the case, which presents a state law claim against a non-diverse defendant. There is no diversity between the parties, nor is there a federal question presented here.

An injured man brought a products liability case in state court in State A against the manufacturer of a drug that the man believes caused him injuries and against the retailer from whom the man bought the drug. The injured man is a citizen in State A. The manufacturer is a corporation that is incorporated in State B, has its sole manufacturing plant in State C, and holds the meetings of its board of directors in State A where they direct and control the corporation's activities. The retailer is a limited liability company that has its principal place of business in State A, but its owner-members are citizens of State E. The injured man seeks $100,000 in compensatory damages. The manufacturer and the retailer timely removed the case to the federal district court in State A. The injured man timely moved to remand the case back to state court. The district court is extremely busy with criminal cases and civil cases that arise under federal law. Should the district court uphold the removal and retain the case, or remand the case to state court?

The district court should remand the case to state court because the injured man is not diverse from both the manufacturer and the retailer, and the court lacks any other basis of subject-matter jurisdiction Here the parties are not completely diverse from one another because the injured man and the manufacturer are both citizens of State A. The plaintiff is domiciled in State A. A corporation is domiciled in both the state of incorporation and the state of its principal place of business. A corporation's principal place of business is typically where it is headquartered. Therefore, the manufacturing corporation is domiciled in both State B, where it is incorporated, and State A, where its board of directors sits. An unincorporated association, such as an LLC, however, has different rules to determine its domicile. An unincorporated association is domiciled in every state in which its members are citizens. Therefore, the drug retailer is domiciled in State E.

A man owned property that he used as his residence. The man received a loan, secured by a mortgage on the property, from a bank. Later, the man defaulted on the loan. The bank then brought an appropriate action to foreclose the mortgage, was the sole bidder at the judicial sale, and received title to the property as a result of the foreclosure sale. Shortly after the foreclosure sale, the man received a substantial inheritance. He approached the bank to repurchase the property, but the bank decided to build a branch office on the property and declined to sell. If the man prevails in an appropriate action to recover title to the property, what is the most likely reason?

The jurisdiction provides for a statutory right of redemption If the man recovers title to the property, it would be because the jurisdiction provides a statutory right of redemption. A jurisdiction may, by statute, provide a statutory right of redemption, which sets out an additional time period after the foreclosure sale during which the prior mortgagor and perhaps others have the option to pay a certain sum of money and redeem the title to the property. The right arises only by statute and only after there has been a foreclosure of the mortgage. If the jurisdiction provides a statutory right of redemption, it does not matter whether the property being redeemed is residential, commercial, or another type of property, unless the statute so notes. Thus, A is incorrect.

A seller and a buyer entered into a written contract for the sale and purchase of land which was complete in all respects except that no reference was made to the quality of title to be conveyed. Which of the following will result?

The seller will be required to convey a marketable title The warranty of marketable title is implied in all land sales contracts. This warranty requires the seller to provide a marketable title to the buyer on (but not before) the closing date. To be considered "marketable," the title must be free of encumbrances, such as mortgages, restrictions, covenants, easements, or other limiting provisions that are not explicitly identified in the contract. Further, in establishing whether an encumbrance exists with regard to local codes or ordinances, it is important to distinguish between building codes and zoning laws. A property being in violation of the building code is not considered an encumbrance, and title to that building is marketable. In contrast, where a property is in violation of zoning laws, the violation is considered an encumbrance and the title is unmarketable. In this case, the warranty of marketable title will be implied in the seller and buyer's contract.

While waiting in line to open an account with a bank, a customer read a poster on the bank's wall that said, "New Customers! $25 FOR 5 MINUTES. If you stand in line for more than five minutes, we will pay you $25! We like happy customers!" The customer started timing his wait and just as five minutes was about to pass, the bank manager tore the poster down and announced, "The $25 stand-in-line promotion is over." The customer waited in line for 10 more minutes before being served. Does the customer have a claim against the bank for $25?

Yes, because the bank could not revoke its offer once the customer had commenced performance The Restatement (Second) of Contracts § 45 provides that where an offer invites acceptance by performance, the offeree's beginning of performance creates an option contract which precludes the offeror from revoking its offer. Because the customer had begun his wait before the poster was taken down and because he ended up standing in line for more than five minutes, the bank should be liable for the $25.

Congress passes an act requiring that all owners of bicycles in the United States register them with a federal bicycle registry. The purpose of the law is to provide reliable evidence of ownership to reduce bicycle theft. No fee is charged for the registration. Although most stolen bicycles are kept or resold by the thieves in the same cities in which the bicycles were stolen, an increasing number of bicycles are being taken to cities in other states for resale. Is this act of Congress constitutional?

Yes, because Congress could determine that in inseverable aggregates bicycle thefts affect interstate commerce. The Commerce Clause empowers Congress to regulate economic or commercial intrastate activity that, in the aggregate, has a substantial effect on interstate commerce. Because the cumulative resale of stolen bikes could rationally affect interstate commerce, the act is constitutional.

An automobile retailer had an adult daughter who needed a car in her employment but had only $3,000 with which to buy one. The retailer wrote to his daughter, "Give me your $3,000 and I'll give you the car on our lot that we have been using as a demonstrator." The daughter thanked her father and paid him the $3,000. As both the retailer and his daughter knew, the demonstrator was reasonably worth $10,000. After the daughter had paid the $3,000, but before the car had been delivered to her, one of retailer's sales staff sold and delivered the same car to a customer for $10,000. Neither the salesperson nor the customer was aware of the transaction between the retailer and his daughter. Does the daughter, after rejecting a tendered return of the $3,000 by her father, have an action against him for breach of contract?

Yes, because the retailer's promise was supported by bargained-for consideration In order to be enforceable, a promise must be supported by bargained-for consideration. Consideration is bargained for if it is sought by the promisor in exchange for the promise. The retailer's promise to give his daughter the car was supported by consideration because the promise was conditioned upon the daughter's promise to pay her father $3,000.

When a mother visited a bowling alley to participate in the weekly bowling league competition held there, she brought her 2 year-old son along and left him in a nursery provided by the bowling alley for the convenience of his customers. The children in the nursery were normally supervised by three attendants, but at this particular time, as the mother knew, there was only one attendant present to care for about twenty children of assorted ages. About thirty minutes later, while the attendant was looking the other way, the son suddenly started to cry. The attendant found him lying on his back, picked him up, and called his mother. It was later discovered that the son had suffered a skull fracture. If a claim is asserted against the bowling alley on the son's behalf, will the son prevail?

No, because the bowling alley and its employees exercised reasonable care to assure the son's safety Here, there is no intentional tort and babysitting is not a source of strict liability. Instead, negligence is the basis for this claim so the first step is determining the duty of care owed. The mother was a business invitee and was owed a duty of ordinary care by the bowling alley. The son will also be classified as a business invitee because he accompanied his mother, even though he is not there to conduct business himself. There is no special relationship here that would heighten the level of duty owed to the mother or son. The bowling alley and nursery have only a reasonable duty to use ordinary care. There is nothing in the facts to suggest that the nursery did not use ordinary care. Children often fall down and get hurt on their own accord. Even with more workers, or with a worker watching only the son, it would not prevent him from falling down and getting hurt. This answer provides the appropriate standard of care and the correct result.

A plaintiff suffered from a serious, though not immediately life-threatening impairment of his circulatory system. The plaintiff's cardiologist recommended a cardiac bypass operation and referred the plaintiff to a surgeon. The surgeon did not inform the plaintiff of the 2% risk of death associated with this operation. The surgeon defended his decision not to mention the risk statistics to the plaintiff because the plaintiff "was a worrier and it would significantly lessen his chance of survival to be worried about the nonsurvival rate." The surgeon successfully performed the bypass operation and the plaintiff made a good recovery. However, when the plaintiff learned of the 2% risk of death associated with the operation, he was furious that the surgeon had failed to disclose this information to him, saying that he would have refused the operation if he had known of the risk. If the plaintiff asserts a claim against the surgeon based on negligence, will the plaintiff prevail?

No, because the operation was successful and the plaintiff suffered no harm Negligence (including medical malpractice) requires proof of duty, breach, causation, and damages. It is not a dignitary tort. If the plaintiff did not suffer an actual injury, he cannot prevail in an action for negligence against his doctor.

In 2006, a utility company constructed a new plant for the generation of electricity. The plant burns lignite, a low-grade fuel which is available in large quantities. Although the plant was constructed in accordance with the best practicable technology, the plant emits a substantial quantity of invisible fumes. The only way the utility company can reduce the fumes is by the use of scrubbing equipment that would cost $50,000,000 to install and would increase the retail price of generated electricity by 50 percent while reducing the volume of fumes by only 20 percent. Because of the expense of such equipment and its relative ineffectiveness, no other generating plants burning lignite use such equipment. The plant was located in a sparsely settled rural area, remote from the large city served by the utility company. A farmer owned a farm adjacent to the plant. He had farmed the land for forty years and lived on the premises. The prevailing winds carried fumes from the new plant over the farmer's land. His 2006 crop was less than half the average size of this crop over the five years immediately preceding the construction of the plant. It can be established that the fumes caused the crop reduction. The farmer's hay fever, from which he had long suffered, became worse in 2006. Physicians advised him that the lignite fumes were affecting it and that serious lung disease would soon result unless he moved away from the plant. He did so, selling his farm at its reasonable market value, which was then $10,000 less than before the construction of the plant. If the farmer asserts a claim based on negligence against the utility company for crop damages, will he prevail?

No, because the utility company was not negligent The facts clearly indicate that the utility company met the current industry standard of care and to do more was unduly burdensome. The plant was constructed in accordance with the best practicable technology and was located in a sparsely populated area. Because the utility company met its burden of care, the farmer will not prevail in a negligence action for his property damage.

The owner in fee simple of Brownacre conveyed Brownacre by quitclaim deed to her daughter who paid no consideration for the conveyance. The deed was never recorded. About a year after the delivery of the deed, the grantor decided that this gift had been ill-advised. She requested that her daughter destroy the deed, which the daughter dutifully and voluntarily did. Within the month following the destruction of the deed, the grantor and the daughter were killed in a common disaster. Each of the successors in interest claimed title to Brownacre. In an appropriate action to determine the title to Brownacre, who will be the probable owner?

The daughter, because the deed was merely evidence of her title, and its destruction was insufficient to cause title to pass back to the grantor The only requirements for transferring legal title in land are execution and delivery of the deed. Execution occurs when the grantor signs the deed. Delivery is effected by proving intent to pass title, even if the grantor never physically gives the title document to the grantee. Once title has been conveyed, the grantee can return it by executing and delivering a new deed back to the grantor. Here, the owner intended to pass title to the daughter and physically delivered a deed to the daughter. The daughter accepted. The owner's execution and delivery of the deed, along with the daughter's acceptance, transferred title to the daughter. The destruction of the deed has no effect, as the deed is merely evidence of title, not the actual title. Therefore, the daughter held title at the time of her death.

Two companies each manufacture pesticide. Their plants are located along the same river. During a specific 24-hour period, each plant discharged pesticide into the river. Both plants were operated negligently and such negligence caused the discharge of pesticide into the river. A rancher operated a cattle ranch downstream from the companies' plants. The rancher's cattle drank from the river and were poisoned by the pesticide. The amount of the discharge from either plant alone would not have been sufficient to cause any harm to the rancher's cattle. If the rancher asserts a claim against the two companies, what, if anything, will the rancher recover?

The entire amount of the rancher's damages, jointly and severally, from the two companies The rancher's damages resulted from the acts of two independent companies, neither of which alone would have been sufficient to poison the cattle, and in circumstances that make it impossible to determine if either defendant alone caused the injury. This is a concurrent cause of harm, and majority courts will hold that where the defendant's negligent act unites with another event, the acts of both defendants will be considered the cause of at least part of the harm, and the defendants will be held jointly and severally liable. The effect is that both companies are liable for the entire amount of the rancher's damages.

A newly enacted federal statute appropriates $100 million in federal funds to support basic research by universities located in the United States. The statute provides that "the ten best universities in the United States" will each receive $10 million. It also provides that "the ten best universities" shall be "determined by a poll of the presidents of all the universities in the nation, to be conducted by the United States Department of Education." In responding to that poll, each university president is required to apply the well-recognized and generally accepted standards of academic quality that are specified in the statute. The provisions of the statute are inseverable. Which of the following statements about this statute is correct?

The statute is constitutional, because Congress has plenary authority to determine the objects of its spending and the methods used to achieve them, so long as they may reasonably be deemed to serve the general welfare and do not violate any prohibitory language in the Constitution Congress's spending power is plenary, and Congress can choose how it spends its money so long as that choice can reasonably be said to serve the general welfare and not violate any prohibition in the Constitution.

A homeowner hired an arsonist to set fire to the homeowner's house so that the homeowner could collect the insurance proceeds from the fire. After pouring gasoline around the house, the arsonist lit the fire with his cigarette lighter and then put the lighter in his pocket. As the arsonist was standing back admiring his work, the lighter exploded in his pocket. The arsonist suffered severe burns to his leg. After finding out that the explosion was caused by a manufacturing defect in the lighter, the arsonist brought an action against the manufacturer of the lighter based on strict product liability. Under applicable law, the rules of pure comparative fault apply in such actions. Will the arsonist prevail?

Yes, because the lighter exploded because of a defect caused by a manufacturing error Don't be fooled by unsympathetic facts. The arsonist was using a lighter for its intended purpose, which was to create a small flame suitable for lighting cigarettes. It was foreseeable that the lighter would also be stored in a user's clothing because it was designed to be portable as part of its purpose and utility in lighting the cigarettes. The arsonist was merely storing the lighter in his pocket when it exploded, which had no causal connection to the fire he had just started. Therefore, despite the fact that moments before, the lighter had been used to start an arsonist's fire, the product was defective and the arsonist may recover in a claim for strict liability from the manufacturer. Strict liability can be imposed upon the manufacturer for the sale of any product that is in a defective or unreasonably dangerous condition and that results in an injury to the user.

The vaccination of children against childhood contagious diseases (such as measles, diphtheria and whooping cough) has traditionally been a function of private doctors and local and state health departments. Because vaccination rates have declined in recent years, especially in urban areas, the President proposes to appoint a Presidential Advisory Commission on Vaccination which would be charged with conducting a national publicity campaign to encourage vaccination as a public health measure. No federal statute authorizes or prohibits this action by the President. The activities of the Presidential Advisory Commission on Vaccination would be financed entirely from funds appropriated by Congress to the Office of the President for "such other purposes as the President may think appropriate." May the President constitutionally create such a commission for this purpose?

Yes, because this action is within the scope of executive authority vested in the President by the Constitution, and no federal statute prohibits it Setting up an advisory commission to encourage vaccination as a public health measure is within the scope of executive authority vested in the President by the Constitution. And, no federal statute prohibits its creation so that even if Congress shares the power to legislate in this area, it has not prohibited executive branch involvement. Thus, the commission is constitutional.

After waiting until all the customers had left, a man entered a small grocery store just before closing time. He went up to the lone clerk in the store and said, "Hand over all the money in the cash register or you will get hurt." The clerk fainted and struck his head on the edge of the counter. As the man went behind the counter to open the cash register, two customers entered the store. The man ran out before he was able to open the register drawer. On this evidence the man could be convicted of

attempted robbery Robbery is the taking, by force or threat of force, the personal property of another with the intent to permanently deprive the owner of the property. In the question, the man was unable to take possession of any personal property of another and thus, did not commit the crime of robbery. To be convicted of attempted robbery, a person must have the intent to commit a robbery and take a substantial step toward the commission of the offense. The man did, by threatening the clerk and going behind the counter, take substantial steps toward the commission of the offense of robbery, and did have the intent to commit the robbery, so he should be convicted of attempted robbery.

A teenager's high school teacher told her that she was going to receive a failing grade in history, which would prevent her from graduating. Furious, she reported to the principal that the teacher had fondled her, and the teacher was fired. A year later, still unable to get work because of the scandal, the teacher committed suicide. The teenager, remorseful, confessed that her accusation had been false. If the teenager is charged with manslaughter, her best defense would be that she

committed no act that proximately caused the teacher's death The teacher committed suicide, and the teenager committed no act that was a proximate cause of the teacher's death. A defendant will only be held criminally responsible for the foreseeable consequences of her actions. The suicide by the teacher was not a foreseeable consequence of the teenager's lying.

The Federal Computer Abuse Act establishes the Federal Computer Abuse Commission, authorizes the Commission to issue licenses for the possession of computers on terms that are consistent with the purposes of the act, and makes the unlicensed possession of a computer a crime. The provisions of the Federal Computer Abuse Act are inseverable. A user applied to the Federal Computer Abuse Commission for a license to possess a computer. The Commission held, and the user participated in, a trial- type proceeding on the user's license application. In that proceeding it was demonstrated that the user repeatedly and intentionally used computers to introduce secret destructive computer programs (computer viruses) into electronic data banks without the consent of their owners. As a result, the Commission denied the user's application for a license. The license denial was based on a Commission rule authorized by the Computer Abuse Act that prohibited the issuance of computer licenses to persons who had engaged in such conduct. Nevertheless, the user retained and continued to use his computer. He was subsequently convicted of the crime of unlicensed possession of a computer. On appeal, he challenges the constitutionality of the licensing provision of the Federal Computer Abuse Act. In this case, the reviewing court would probably hold that act to be

constitutional, because Congress may use the authority vested in it by the commerce clause to regulate the possession of computers and the provisions of this act do not violate any prohibitory provision of the Constitution One of Congress's most important powers is its power to regulate commerce among the several states. There are four categories of activities which Congress can constitutionally regulate under the Commerce Clause: (1) channels of interstate commerce; (2) instrumentalities of interstate commerce; (3) articles moving in interstate commerce; and (4) activities having a substantial effect on interstate commerce. Therefore, Congress has broad power under the Constitution to regulate items in the stream of commerce including the possession of computers. Because this licensing act doesn't violate any prohibitory provision of the Constitution, it is constitutional.

Three states, East Winnetka, Midland, and West Hampton, are located next to one another in that order. The states of East Winnetka and West Hampton permit the hunting and trapping of snipe, but the state of Midland strictly forbids it in order to protect snipe, a rare species of animal, from extinction. The state of Midland has a state statute that provides "Possession of snipe traps is prohibited. Any game warden finding a snipe trap within the state shall seize and destroy it." Snipe traps cost about $15 each. An ardent snipe trapper is a resident of West Hampton. She drove her car to East Winnetka to purchase a new snipe trap from a manufacturer there. In the course of her trip back across Midland with the trap in her car, the snipe trapper stopped in a Midland state park to camp for a few nights. While she was in that park, a Midland game warden saw the trap, which was visible on the front seat of her car. The warden seized the trap and destroyed it in accordance with the Midland statute after the trapper admitted that the seized item was a prohibited snipe trap. No federal statutes or federal administrative regulations apply. The snipe trapper can demonstrate that common carriers are permitted to transport snipe traps as cargo across Midland for delivery to another state and that in practice the Midland statute is enforced only against private individuals transporting those traps in private vehicles. If the trapper challenges the application of the Midland statute to her on the basis only of a denial of equal protection, the application of the statute will probably be found

constitutional, because there is a rational basis for differentiating between the possession of snipe traps as interstate cargo by common carriers and the possession of snipe traps by private individuals Because the statute does not classify on any suspect grounds, rational basis is the test that would apply. It is rational to differentiate between common carriers transporting snipe traps and individuals possessing them, the statute would be constitutional.

The legislature of a particular state enacted a statute requiring that all law enforcement officers in that state be citizens of the United States. An alien, lawfully admitted to permanent residency five years before the enactment of this statute, sought employment as a forensic pathologist in the state coroner's office. He was denied such a job solely because he was not a citizen. The alien thereupon brought suit in federal district court against appropriate state officials seeking to invalidate this citizenship requirement on federal constitutional grounds. The strongest ground upon which to attack this citizenship requirement is that it

denies an alien the equal protection of the laws guaranteed by the Fourteenth Amendment The Equal Protection Clause requires that classifications based on alienage be narrowly tailored to promote a compelling state interest. Because that is such a difficult test to satisfy, this is the strongest argument.

A grand jury indicted the defendant on a charge of arson, and a valid warrant was issued for her arrest. A police officer arrested the defendant and informed her of what the warrant stated. However, hoping that the defendant might say something incriminating, he did not give her Miranda warnings. He placed her in the back seat of his patrol car and was driving her to the police station when she said, "Look, I didn't mean to burn the building; it was an accident. I was just burning some papers in a wastebasket." At the station, after being given Miranda warnings, the defendant stated that she wished to remain silent and made no other statements. The defendant moved to suppress the use of her statement to the police officer as evidence on two grounds: first, that the statement was acquired without giving Miranda warnings, and second, that the police officer had deliberately elicited her incriminating statement after she was in custody. As to the defendant's motion to suppress, the court should

deny the motion Miranda warnings need to be given only if the person is subject to custodial interrogation. The defendant was in custody pursuant to the warrant; however, she was never subject to interrogation. Her voluntary statement in the vehicle to the officer should not be suppressed on the ground that the officer failed to give her the Miranda warnings. Merely hoping for incriminating statements is insufficient action by the officer to be considered an interrogation for Miranda purposes. The officer did not ask any questions, he did not deliberately elicit any statement, and the statement was knowingly and voluntarily made.

A defendant was driving his automobile at a legal speed in a residential zone. A child darted out in front of him and was run over and killed before the defendant could prevent it. The defendant's driver's license had expired three months previously; the defendant had neglected to check when it was due to expire. Driving without a valid license is a misdemeanor in the jurisdiction. On a charge of manslaughter, the defendant should be found

not guilty, because the offense was not the proximate cause of the death For the defendant to be found guilty of manslaughter, a proximate cause between the illegal act and the resulting death of the victim must exist. This causation must be more than simple cause-in-fact; it must be somehow foreseeable from the illegal action taken. In this case, the fact the defendant did not possess a valid license was not a causal factor in the death of the child. death

A man and a woman planned to break into a federal government office to steal food stamps. The man telephoned an associate one night and asked whether the associate wanted to buy some "hot" food stamps. The associate, who understood that "hot" meant stolen, said, "Sure, bring them right over." The man and the woman then successfully executed their scheme. That same night they delivered the food stamps to the associate, who bought them for $500. The associate did not ask when or by whom the stamps were stolen. All three were arrested. The man and the woman entered guilty pleas in federal court to a charge of larceny in connection with the theft. The associate was brought to trial in the state court on a charge of conspiracy to steal food stamps. On the evidence stated, the associate should be found

not guilty, because, although the associate knew the stamps were stolen, he neither helped to plan nor participated or assisted in the theft Conspiracy is an agreement between two or more people to commit a crime. A charge of conspiracy requires an agreement that a crime be committed and that at least two of the co-conspirators intend that the crime be committed. The associate did not agree to commit the theft, nor did he help to plan, participate or assist in the theft of the food stamps. Although the associate may be guilty of conspiracy to receive stolen property, he did not agree to steal the food stamps. Therefore, he should not be found guilty of conspiracy to steal food stamps.

A child was the illegitimate, unacknowledged child of the decedent. The decedent died intestate, leaving neither spouse nor any children other than the illegitimate child. The state's law of intestate succession provides that an unacknowledged illegitimate child may not inherit his father's property. The spouse, all other blood relations, and the state are preferred as heirs over the unacknowledged illegitimate child. The illegitimate child filed suit in an appropriate court alleging that the state statute barring an illegitimate child from sharing in a parent's estate is invalid and that he should be declared a lawful heir to his father's estate. In challenging the validity of the state statute, the illegitimate child's strongest argument would be that

preferring collateral relatives and even the state over unacknowledged children is not substantially related to an important government interest, and, therefore, the law violates the Equal Protection Clause The state statute classifies on the basis of legitimacy, so it will be subject to intermediate scrutiny. Thus, the strongest argument is that the classification is not substantially related to an important government interest and thus violates the Equal Protection Clause of the Fourteenth Amendment.

A light company is the sole distributor of electrical power in a city. The company owns and maintains all of the electric poles and equipment in the city. The company has complied with the National Electrical Safety Code, which established minimum requirements for the installation and maintenance of power poles. The code has been approved by the federal and state governments. The company has had to constantly replace insulators on its poles because unknown people repeatedly shoot at and destroy them. This causes the power lines to fall to the ground. The light company did not use other reasonable means to secure the lines to the poles. On an occasion when the lines had fallen, a 5-year-old child wandered out of his yard, intentionally touched a downed wire, and was seriously burned. If a claim on the child's behalf is asserted against the light company, the probable result is that the child will

recover, because the light company could have taken reasonable steps to prevent the lines from falling when the insulators were destroyed The light company has equipment located in a public area, so it must use reasonable care to maintain that equipment in such a way as to prevent danger to the public. Because the light company failed to take a reasonable precaution to a known danger, the child will recover.

Several years ago, a carpenter purchased Goldacre, financing a large part of the purchase price by a loan from a bank that was secured by a mortgage. The carpenter made the installment payments on the mortgage regularly until last year. Then the carpenter persuaded a friend to buy Goldacre, subject to the mortgage to the bank. They expressly agreed that the friend would not assume and agree to pay the carpenter's debt to the bank. The carpenter's mortgage to the bank contained a due-on-sale clause stating, "If Mortgagor transfers his/her interest without the written consent of Mortgagee first obtained, then at Mortgagee's option the entire principal balance of the debt secured by this Mortgage shall become immediately due and payable." However, without seeking the bank's consent, the carpenter conveyed Goldacre to the friend, the deed stating in pertinent part ". . . subject to a mortgage to the bank [giving details and recording data]." The friend took possession of Goldacre and made several mortgage payments, which the bank accepted. Now, however, neither the friend nor the carpenter has made the last three mortgage payments. The bank has brought an appropriate action against the friend for the amount of the delinquent payments. In this action, judgment should be for

the friend, because she did not assume and agree to pay the carpenter's mortgage debt The mortgagor of a property is generally free to transfer title to that property BUT: 1) the mortgagor remains personally liable on the mortgage; and 2) all subsequent grantees take the property subject to the mortgage. Further, subsequent grantees do not become personally liable on the mortgage unless they explicitly assume the mortgage. Therefore, the mortgagor will always be personally liable on the mortgage, all subsequent grantees who assume the mortgage will be personally liable on it, and all subsequent grantees (whether they assume the mortgage or not) can lose the property through foreclosure if the mortgage is not paid. The facts state that the friend did not assume the mortgage, so she has no personal liability to the bank with regard to mortgage payments. Although she voluntarily made such payments for a few months, she was under no obligation to do so. Therefore, answer A is correct. The bank, however, has the right to foreclose on the mortgage if the payments are not made, so the friend stands to lose the property unless either she or the carpenter makes the payments. Further, the carpenter's conveyance without the bank's permission allows the bank, pursuant to the due-on-sale clause, to demand immediate payment of the entire outstanding mortgage balance.

A landowner owned Blueacre, a valuable tract of land located in York County. The owner executed a document in the form of a warranty deed of Blueacre, which was regular in all respects except that the only language designating the grantees in each of the granting and habendum clauses was: "The leaders of all the Protestant Churches in York County." The instrument was acknowledged as required by statute and promptly and properly recorded. The owner told his lawyer, but no one else, that he had made the conveyance as he did because he abhorred sectarianism in the Protestant movement and because he thought that the leaders would devote the asset to lessening sectarianism. The owner died suddenly and unexpectedly a week later, leaving a will that bequeathed and devised his entire estate to a friend. After probate of the will became final and the administration on the owner's estate was closed, the friend instituted an appropriate action to quiet title to Blueacre and properly served as defendant each Protestant church situated in the county. The only evidence introduced consisted of the chain of title under which the owner held, the probated will, the recorded deed, the fact that no person knew about the deed except the owner and his lawyer, and the conversation the owner had with his lawyer described above. In such action, judgment should be for

the friend, because there is inadequate identification of the grantees in the deed A deed must unambiguously identify the land and the parties to transfer title. A grant to "the leaders of all the Protestant Churches in York County" is simply too broad and vague to adequately identify the intended grantees. Therefore, the deed will not transfer title, and the friend will take Blueacre under the will.

A college student purchased a large bottle of No-Flake dandruff shampoo, manufactured by a shampoo company. The box containing the bottle stated in part: "CAUTION--Use only 1 capful at most once a day. Greater use may cause severe damage to the scalp." The college student read the writing on the box, removed the bottle, and threw the box away. The college student's roommate asked to use the No-Flake, and college student said, "Be careful not to use too much." The roommate thereafter used No-Flake twice a day, applying two or three capfuls each time, notwithstanding the label statement that read: "Use no more than one capful per day. See box instructions." The more he used No- Flake, the more inflamed his scalp became, the more it itched, and the more he used. After three weeks of such use, the roommate finally consulted a doctor who diagnosed his problem as a serious and irreversible case of dermatitis caused by excessive exposure to the active ingredients in No-Flake. These ingredients are uniquely effective at controlling dandruff, but there is no way to remove a remote risk to a small percentage of persons who may contract dermatitis as the result of applying, for prolonged periods of time, amounts of No-Flake substantially in excess of the directions. This jurisdiction adheres to the traditional common-law rules pertaining to contributory negligence and assumption of risk. Based upon the foregoing facts, if the roommate sues the shampoo company to recover damages for his dermatitis, his most promising theory of liability will be that the No-Flake shampoo

was inadequately labeled to warn of its dangers An inadequate warning can function as a type of design defect claim where the product is in a defective condition or is unreasonably dangerous to the user based on a design or manufacturing defect. A product is unreasonably dangerous when it is dangerous beyond the expectations of the ordinary consumer because of a departure from the intended design. A type of design defect is an inadequate warning which does not provide users with clear and complete warnings of dangers, which may not be apparent. Here, the shampoo did not depart from the intended design. Further, the product was substantially altered by the time it reached the roommate, because the college student had disposed of the packaging. However, although the shampoo directed users to use no more than one capful per day, it did not warn users of the minimal risk of developing dermatitis from frequent overuse. Therefore, the roommate's best argument would be that the shampoo was inadequately labeled to warn of its dangers.

Two lawyers work as partners in a small town that has only one other lawyer in it. The partners do a substantial amount of personal injury work. A client was severely and permanently injured in an automobile collision. The client employed the partners to represent her in obtaining damages from the motorist for her injuries. At the time she employed the partners, the statute of limitations on her claim had six weeks to run. The complaint was prepared but not filed. Each partner thought that the other partner would file the complaint. The statute of limitations ran on the client's claim against the motorist. The client has filed suit against the partners for negligence. That case is on trial with a jury in a court of general jurisdiction. In addition to proving that the partners were negligent, the client must establish, as a minimum, that she

would have, but for her lawyers' negligence, recovered from the motorist Unlike ordinary negligence, the plaintiff in a legal malpractice claim must prove that: (i) the lawyer's misconduct or lack of action caused her loss; and (ii) that, had the lawyer met the standard of care, the plaintiff would have won her claim or defense and been eligible to receive a specifically defined result. This situation is often called the "trial within a trial" burden. Consequently, while answers B, C, and D are all partially correct, they do not go far enough in establishing the client's minimum burden of proof in a claim for legal malpractice/negligence.

By warranty deed, a woman conveyed Blackacre to her friend and her neighbor "as joint tenants with right of survivorship." The friend and neighbor are not related. The friend conveyed all her interest to her boyfriend by warranty deed and subsequently died intestate. Thereafter, the neighbor conveyed to his girlfriend by warranty deed. There is no applicable statute, and the jurisdiction recognizes the common-law joint tenancy. Title to Blackacre is in

the girlfriend and the boyfriend The facts state that the property was held by the friend and neighbor in joint tenancy. Where one joint tenant makes an inter vivos conveyance of their interest in a property, a severance of the joint tenancy occurs and the interest transferred is that of a tenant in common. As soon as the friend made an inter vivos conveyance to her boyfriend, the joint tenancy was severed, and he and the neighbor became tenants in common. As such, the neighbor's conveyance to his girlfriend was likewise a transfer of an interest as a tenant in common. Therefore, the girlfriend and boyfriend hold title as tenants in common

A driver was driving his car near a homeowner's house when the homeowner's child darted into the street in front of the driver's car. As the driver swerved and braked his car to avoid hitting the child, the car skidded up into the homeowner's driveway and stopped just short of the homeowner, who was standing in the driveway and had witnessed the entire incident. The homeowner suffered from serious emotional distress from witnessing the danger to his child and to himself. Neither the homeowner nor his property was physically harmed. If the homeowner asserts a claim for damages against the driver but is unable to establish that the driver was negligent, will the homeowner still be able to prevail?

No, because the homeowner failed to show that the driver was negligent. The homeowner is most likely bringing a claim for negligent infliction of emotional distress against the driver. A stand-alone claim for emotional distress requires a negligent act (1) that results in a personally witnessed injury to a close family member (such as parent & child), or (2) where the plaintiff had been within the "zone of danger" for injury himself. Certainly in this case, the homeowner witnessed the near injury to his own child and then his own near injury. In both cases, however, there was no actual physical injury. And, with the driver merely reacting to the sudden presence of a child in the road, there is also the key issue of whether the driver was negligent at all. Unless there is a negligent act, there cannot be a recovery, therefore B is incorrect.

A college student purchased a large bottle of No-Flake dandruff shampoo, manufactured by a shampoo company. The box containing the bottle stated in part: "CAUTION--Use only 1 capful at most once a day. Greater use may cause severe damage to the scalp." The college student read the writing on the box, removed the bottle, and threw the box away. The college student's roommate asked to use the No-Flake, and college student said, "Be careful not to use too much." The roommate thereafter used No-Flake twice a day, applying two or three capfuls each time, notwithstanding the label statement that read: "Use no more than one capful per day. See box instructions." The more he used No-Flake, the more inflamed his scalp became, the more it itched, and the more he used. After three weeks of such use, the roommate finally consulted a doctor who diagnosed his problem as a serious and irreversible case of dermatitis caused by excessive exposure to the active ingredients in No-Flake. These ingredients are uniquely effective at controlling dandruff, but there is no way to remove a remote risk to a small percentage of persons who may contract dermatitis as the result of applying, for prolonged periods of time, amounts of No-Flake substantially in excess of the directions. This jurisdiction adheres to the traditional common-law rules pertaining to contributory negligence and assumption of risk. Based upon the foregoing facts, if the roommate sues the shampoo company to recover damages for his dermatitis, his most promising theory of liability will be that the No-Flake shampoo

was inadequately labeled to warn of its dangers. An inadequate warning can function as a type of design defect claim where the product is in a defective condition or is unreasonably dangerous to the user based on a design or manufacturing defect. A product is unreasonably dangerous when it is dangerous beyond the expectations of the ordinary consumer because of a departure from the intended design. A type of design defect is an inadequate warning which does not provide users with clear and complete warnings of dangers, which may not be apparent. Here, the shampoo did not depart from the intended design. Further, the product was substantially altered by the time it reached the roommate, because the college student had disposed of the packaging. However, although the shampoo directed users to use no more than one capful per day, it did not warn users of the minimal risk of developing dermatitis from frequent overuse. Therefore, the roommate's best argument would be that the shampoo was inadequately labeled to warn of its dangers.

A woman decided to steal a necklace that belonged to her neighbor. She knew where the neighbor kept the necklace because she had been in the neighbor's house on many occasions when the neighbor had taken off the necklace and put it away in a jewelry box in the bathroom. One night, the woman went to the neighbor's house. The neighbor was away, and the house was dark. The woman opened the bathroom window, saw the jewelry box on the counter, and started to climb inside. As her leg cleared the window sill, the neighbor's dog began to bark loudly. Terrified, the woman jumped back outside and fled. The crimes below are listed in descending order of seriousness. What is the most serious crime, if any, committed by the woman?

Burglary The woman is guilty of burglary because she unlawfully entered the neighbor's house at night with intent to commit a felony (larceny). The woman's actions constituted the requisite entry of the neighbor's house. To constitute burglary, it is sufficient if any part of the actor's person intruded, even momentarily, into the structure. Thus it has been held that the intrusion of a part of a hand in opening a window, or the momentary intrusion of part of a foot clearing the window sill, constituted the requisite entry.

A licensor of an education program sued a teacher in federal court, alleging trademark infringement and other wrongs because the teacher continued use of the licensor's programs even after the teacher's license to use the programs expired. To prevail on its infringement claim, the licensor had to prove, among other things, that the teacher used the trademark in a manner likely to confuse consumers. That element turned in part on the similarity between the products and services offered by the licensor and the teacher, and whether consumers were actually confused. The case was tried to a federal judge in a bench trial. The judge held for the licensor on its trademark infringement claim, and entered judgment for the licensor. The teacher then filed a timely notice of appeal. The appeal raised the question of the proper standard of review for the determination that the teacher used the trademark in a manner likely to confuse consumers. Under what circumstances should the federal appeals court reverse the district court's judgment?

If the appellate court concludes that the district court's determination that the teacher used the trademark in a manner likely to confuse consumers was a clearly erroneous fact-finding Findings of fact, whether based on oral or other evidence, must not be set aside unless clearly erroneous. The clearly erroneous standard applies when the court is left with a definite and firm conviction that a mistake has been committed. Here, the determinants of this element of the infringement claim are essentially factual matters, therefore the clearly erroneous standard applies.

On December 15, a lawyer received from a retailer of supplies an offer consisting of its catalog and a signed letter stating, "We will supply you with as many of the items in the enclosed catalog as you order during the next calendar year. We assure you that this offer and the prices in the catalog will remain firm throughout the coming year." Assume that on January 15, having at that time received no reply from the lawyer, the retailer notified the lawyer that effective February 1, it was increasing the prices of certain specified items in its catalog. Is the price increase effective with respect to catalog orders the retailer receives from the lawyer during the month of February?

No, becNo because the retailer is a merchant with respect to office supplies; and its orginal offer, including the price term, was irrevocable throughout the month of February. Under the UCC's firm offer rule, an offer by a merchant for the sale of goods contained in a signed writing which by its terms gives assurance that it is firm will be irrevocable, notwithstanding lack of consideration, for the time stated in the offer, but in no case may the period of irrevocability exceed three months. The retailer is a merchant, because it is a retailer of office supplies. The offer that it sent to the lawyer is a firm offer because it was made by a merchant, is contained in a signed writing, and provides the necessary assurances. The fact that the period of irrevocability exceeds three months does not invalidate the firm offer, but the offer will only be firm for a three-month period. Because, as of January 15, the three-month period had not yet elapsed, the offer is still firm and the retailer is not free to change the prices contained in the December 15 offer.

While driving at a speed in excess of the statutory limit, the defendant negligently collided with another car, and the disabled vehicles blocked two of the highway's three northbound lanes. When the plaintiff approached the scene two minutes later, he slowed his car to see if he could help those involved in the collision. As he slowed, he was rear-ended by the driver of another vehicle. The plaintiff, who sustained damage to his car and was seriously injured, brought an action against the defendant to recover damages. The jurisdiction adheres to the traditional common law rules pertaining to contributory negligence. If the defendant moves to dismiss the action for failure to state a claim upon which relief may be granted, should the motion be granted?

No, because a jury could find that the plaintiff's injury arose from a risk that was a continuing consequence of the defendant's negligence. This question addresses the issue of causation in a negligence action, and causation requires that the defendant's negligence be both the actual and legal cause of the plaintiff's injuries. The defendant is the actual cause of the plaintiff's injuries because but for the defendant's negligent speeding, the plaintiff would not have been injured. Furthermore, it is possible the defendant is the legal cause of the plaintiff's injuries because it is foreseeable that speeding would cause an accident, and that the accident would create a dangerous road condition leading to subsequent accidents.

Insurance is provided in a particular state only by private companies. Although the state insurance commissioner inspects insurance companies for solvency, the state does not regulate their rates or policies. An insurance company charges higher rates for burglary insurance to residents of one part of a county in the state than to residents of another section of the same county because of the different crime rates in those areas. The plaintiff is a resident of the county who was charged the higher rate by the insurance company because of the location of her residence. The plaintiff sues the insurance company, alleging that the differential in insurance rates unconstitutionally denies her the equal protection of the law. Will the plaintiff's suit succeed?

No, because the constitutional guarantee of equal protection of the law is not applicable to the actions of these insurance companies. The Constitution provides for equal protection of the law, which means that it protects individuals from actions by the state (the Fourteenth Amendment) or the federal government (the Fifth Amendment). Equal protection only restricts private action in extremely specific circumstances, none of which are present in this fact pattern.

A famous chef entered into a written agreement with his friend, a well-known interior decorator respected for his unique designs, in which the decorator agreed, for a fixed fee, to design the interior of the chef's new restaurant, and, upon the chef's approval of the design plan, to decorate and furnish the restaurant accordingly. The agreement was silent as to assignment or delegation by either party. Before beginning the work, the decorator sold his decorating business to Newman under an agreement in which the decorator assigned to Newman, and Newman agreed to complete, the chef-decorator contract. Newman, also an experienced decorator of excellent repute, advised the chef of the assignment, and supplied him with information confirming both Newman's financial responsibility and past commercial success. Is the chef obligated to permit Newman to perform the chef-decorator agreement?

No, because the decorator's duties wer of a personal nature, involving his reputation, taste and skill Unless the contract provides otherwise, a contractual duty may be delegated to another unless the other party to the contract has a substantial interest in having the original obligor perform. Typically the other party will have such an interest where the contract is a personal services contract involving fancy, taste and judgment. Although the the chef-decorator contract was silent on the issue of assignability, the facts suggest that the chef would have a substantial interest in having the decorator and no other person design the interior of his new restaurant. Therefore, the decorator's delegation of the duty to Newman without the chef's consent amounts to a breach of contract.

A worker from State A sues a farmer from State B in a state court in State B, asserting a one-count complaint alleging that the farmer violated the worker's rights under Title VII of the federal 1964 Anti-Discrimination Act. The farmer files a notice of removal in the federal court of State B. The worker then files a motion for remand to state court. Should the federal judge grant the worker's motion for remand?

No, because the federal court has jurisdiction under the federal question statute The removal statute allows a defendant to remove a civil action brought in state court if the federal court has federal question jurisdiction. Therefore, because the complaint alleges a violation of the federal 1964 Anti- Discrimination Act, a federal law, the farmer can remove to federal court in State B.

A property owner owns a hotel. When an international charity organization came to town for its annual convention, its members rented 400 of the 500 rooms, and the hotel opened its convention facilities to them. During their convention, the members littered both the inside and the outside of the hotel with debris and bottles. The hotel manager knew that objects were being thrown out of the hotel windows. At his direction, hotel employees patrolled the hallways telling the guests to refrain from such conduct. The owner was out of town and was not aware of the problems which were occurring. During the convention, as a pedestrian walked past the hotel on the sidewalk, he was hit and injured by an ashtray thrown out of a window in the hotel by an unknown person. The pedestrian sued the owner for damages for his injuries. Will the pedestrian prevail in his claim against the owner?

No, because the hotel employees had taken reasonable precautions to prevent such an injury. The injured person was not a guest, but a passerby, so the principle that innkeepers are liable for slight negligence does not apply to this situation. Instead, the pedestrian was a licensee, so the owner had a duty to exercise reasonable care in conducting active operations. Because the hotel employees took reasonable precautions against injury to a passerby, the owner will not be found liable.

A pilot from State A brought suit against a librarian from State B, in a single count complaint filed in State A state court. The librarian sold the pilot an expensive collectible watch through an internet site that generated a valid sales contract. The pilot's complaint alleges that the librarian breached the contract's terms because the librarian sold the pilot a watch that materially varied from the description the librarian had provided. The pilot sought $175,000 in damages. Shortly after selling the watch, the librarian went to State A to vacation with her family. This was the librarian's first time ever in State A. While visiting the state capitol in State A, the librarian was served with the pilot's complaint and a summons in accordance with State A law. The librarian's lawyer files a motion to dismiss for lack of jurisdiction. Will the court grant the motion?

No, because the librarian's physical presence in State A when she was served is sufficient to give the court personal jurisdiction over her A court has personal jurisdiction over a defendant if she is present in the forum state when personally served with process.

An ophthalmologist from State A brought suit against a manufacturer from State B, in a single count complaint filed in State A state court. The manufacturer makes widgets in a small factory and sells them throughout the country through his website. The ophthalmologist purchased a widget from the manufacturer through the manufacturer's website after seeing an advertisement for the widget in a State A newspaper. The ophthalmologist was injured when the widget malfunctioned. In his complaint, the ophthalmologist seeks $85,000 in damages under State A products liability law. State A's long-arm statute authorizes the state to have the full range of constitutionally valid personal jurisdiction. The manufacturer's lawyer files a motion with the court to dismiss for lack of jurisdiction because the manufacturer has never been to State A. Will the court grant the manufacturer's motion to dismiss for lack of jurisdiction?

No, because the manufacturer has purposely availed himself to State A law by advertising and selling his products there The manufacturer has the required minimum contacts with State A. Therefore, an exercise of personal jurisdiction would be fair and reasonable. By advertising and selling his goods in State A, the manufacturer has targeted citizens of State A and purposefully availed himself to the benefits of conducting business within State A. The manufacturer's activity in State A makes it reasonable and foreseeable that he may be called into court there to defend a lawsuit.

Police officers received a tip that illegal drugs were being sold at a certain ground-floor apartment. They decided to stake out the apartment. The stakeout revealed that a significant number of people visited the apartment for short periods of time and then left. A man exited the apartment and started to walk briskly away. The officers grabbed the man and, when he struggled, wrestled him to the ground. They searched him and found a bag of heroin in one of his pockets. After discovering the heroin on the man, the officers decided to enter the apartment. They knocked on the door, which was opened by the woman who lived there. The officers asked if they could come inside, and the woman gave them permission to do so. Once inside, the officers observed several bags of heroin on the living room table. The woman has been charged with possession of the heroin found on the living room table. She has filed a pretrial motion to suppress the heroin on the ground that it was obtained by an illegal search and seizure. Should the woman's motion be granted?

No, because the woman consented to the officers' entry. In the absence of an exception, the Fourth Amendment requires that police have both probable cause and a search warrant before they are able to enter a private dwelling. Here, no such warrant existed. However, police may conduct a valid warrantless search if they have a voluntary and intelligent consent to do so. Here, the facts do not indicate that the police improperly obtained the woman's consent. The woman's consent justified the officers' entry. Once inside, the police properly seized the heroin because it was in plain view. Police may make a warrantless seizure when they are legitimately on the premises and discover evidence that is in plain view that they have probable cause to believe is evidence or a fruit or instrumentality of crime. Here, the woman's consent gave the police legitimate authority to be inside the building, they discovered the heroin on a living room table, and the criminal nature of the heroin was readily apparent.

On December 1, a broker contracted with a collector to sell her one of a certain type of rare coin for $12,000, delivery and payment to occur on the next March 1. To fulfill that contract, and without the collector's knowledge, the broker contracted on January 1 to purchase for $10,000 a specimen of that type of coin from a hoarder, delivery and payment to occur on February 1. The market price of such coins had unexpectedly fallen to $8,000 by February 1, when the hoarder tendered the coin and the broker repudiated. On February 25, the market in such coins suddenly reversed and had stabilized at $12,000 on March 1. The broker, however, had failed to obtain a specimen of the coin and repudiated his agreement with the collector when she tendered the $12,000 agreed price on March 1. Later that day, after learning by chance of the broker's dealing with the collector, the hoarder telephoned the collector and said: "Listen, the broker probably owes me at least $2,000 in damages for refusing wrongfully to buy my coin for $10,000 on February 1 when the market was down to $8,000. But I'm in good shape in view of the market's recovery since then, and I think you ought to get after the so-and-so." If the collector immediately sues the broker for his breach of the broker-hoarder contract, which of the following will the court probably decide?

The broker wins, because the collector, if a beneficiary at all of the broker-hoarder contract, was only an incidental beneficiary A third party acquires standing to enforce a promise only if that party is an intended beneficiary of the promise. A party is an intended beneficiary of a promise if recognition of a right to performance in the beneficiary is appropriate to effectuate the intent of the parties, such as where the promisor must know that the promisee intended to confer a benefit on the beneficiary. Incidental beneficiaries are third parties who unintentionally or inadvertently benefit from a contract; they have no enforceable rights. The collector is only an incidental beneficiary of the broker-hoarder contract because the facts suggest that, in agreeing to sell the rare coin to the broker on February 1, the hoarder did not intend to confer a benefit on the collector.

A new clothing company, about to commence the manufacture of a new clothing line, entered into a written agreement to purchase all of its monthly requirements of a certain elasticized fabric for a period of three years from a fabric company at a specified unit price and agreed upon delivery and payment terms. The agreement also provided that the parties covenant not to assign this contract. The fabric company promptly made an "assignment of the contract" to a textile company. The clothing company subsequently ordered, took delivery of, and paid the fabric company the agreed price ($5,000) for the requirement of the fabric for the first month of its operation. Which of the following accurately states the legal effect of the covenant not to assign the contract?

The fabric company's assignment was a breach of its contract with the clothing company was nevertheless effective to transfer to the textile company the abric company's rights against the clothing company The fabric company breached the contract because it contained an express prohibition of the assignment of rights. However, under UCC § 2-210(4) and §322 of the Restatement (Second) of Contracts, absent circumstances suggesting otherwise, a clause prohibiting the assignment of "the contract" will be construed as barring only the delegation of the assignor's duties. Further, a clause prohibiting the assignment of rights, like the right to payment, generally will NOT bar the assignment of those rights. The assignor has the power but not the right to assign, so if an assignment is made, the obligor has the right to sue for the breach. The assignment of rights to the textile company will be effective, but the clothing company will have the right to sue for breach, as the fabric company breached an express provision of the contract.

A painter, who has been in the painting business for ten years and has a fine reputation, contracts to paint a farmer's barn. The farmer's barn is a standard red barn with a loft. The contract has no provision regarding assignment. If the painter assigns the contract to a contractor, who has comparable experience and reputation, which of the following statements is correct?

The farmer is required to accept performance by the contractor An assignment is a transfer of rights under a contract. Generally, all contractual rights may be assigned unless the contract provides otherwise. A contractual right may be assigned to another unless the other party to the contract has a substantial interest in having the original obligor perform. Typically the other party will have a substantial interest if the contract is a personal services contract involving aesthetic taste and judgment. Although the painter-farmer contract is one for personal services (painting), the item to be painted is a standard barn and the work will be done by a painter of comparable experience and reputation. A delegation, on the other hand, is the transfer of duties under a contract. In this case, because the painter assigned the entire contract, there was both an assignment and delegation of duties. There are no special formalities for an effective delegation. The delegator must only manifest a present intention to make the delegation. Therefore, the painter's delegation of the duty was not a breach, even if it was done without the consent of the farmer. If the farmer refuses performance by the contractor, he will be in breach of contract.

A painter, who has been in the painting business for ten years and has a fine reputation, contracts to paint a farmer's barn. The farmer's barn is a standard red barn with a loft. The contract has no provision regarding assignment. If the painter assigns the contract to a contractor, who has comparable experience and reputation, which of the following statements is correct?

The farmer is required to accept performance by the contractor An assignment is a transfer of rights under a contract. Generally, all contractual rights may be assigned unless the contract provides otherwise. A contractual right may be assigned to another unless the other party to the contract has a substantial interest in having the original obligor perform. Typically the other party will have a substantial interest if the contract is a personal services contract involving aesthetic taste and judgment. Although the painter-farmer contract is one for personal services (painting), the item to be painted is a standard barn and the work will be done by a painter of comparable experience and reputation. A delegation, on the other hand, is the transfer of duties under a contract. In this case, because the painter assigned the entire contract, there was both an assignment and delegation of duties. There are no special formalities for an effective delegation. The delegator must only manifest a present intention to make the delegation. Therefore, the painter's delegation of the duty was not a breach, even if it was done without the consent of the farmer. If the farmer refuses performance by the contractor, he will be in breach of contract.

A pharmacist filed a lawsuit against her employer, a hospital, in federal court in State A, claiming that the hospital violated the pharmacist's rights under Title VII's employment laws during the two weeks the pharmacist worked for the hospital. After two days of testimony, the jury returned a verdict for the pharmacist and awarded her $2,750,000 in compensatory damages. Under federal law, a judge who believes compensatory damages are so excessive as to "shock the conscience" can offer the plaintiff the choice between a new trial or remittitur of the excessive damages. Under State A law, judges can order remittitur if the court thinks damages are "excessive." Which of the following is true?

The hospital should submit both a motion for new trial and, in the alternative, a motion for remittitur, on the ground that the jury's damage award "shocks the conscience" This lawsuit arises under federal question jurisdiction. Therefore, this is not a diversity jurisdiction lawsuit and there are no Erie issues. Federal law alone governs. Therefore "shocks the conscience," not "excessive," is the appropriate standard to apply.

In a writing signed by both parties on December 1, a man agreed to buy from a distributor a gasoline engine for $1,000, delivery to be made on the following February 1. Through a secretarial error, the writing called for delivery on March 1, but neither party noticed the error until February 1. Before signing the agreement, the man and the distributor orally agreed that the contract of sale would be effective only if the man notified the distributor in writing no later than January 2 that the man had arranged to resell the engine to a third person. Otherwise, they agreed orally, "There is no deal." On December 15, the man entered into a contract with a mechanic to resell the engine to the mechanic at a profit. The man did not give the distributor notice of the resale until January 25, and the distributor received it by mail on January 26. In the meantime, the value of the engine had unexpectedly increased about 75% since December 1, and the distributor renounced the agreement.

The man's not giving written notice by January 2 of his resale was a failure of a condition precedent to the existence of a contract A condition is an event that is not certain to occur, which must occur, unless excused, before performance under a contract becomes due. The man and the distributor agreed that each party's obligation under the contract of sale was expressly conditioned upon the man giving written notice of resale to the distributor on or before January 2. Since such notice was not given until January 26, any obligation that the distributor undertook under the contract of sale was excused due to the nonoccurrence of the condition. Although the express condition was not written into the contract, it was orally agreed to by both parties as being a condition upon which the contract was made. Thus, although it was not included in writing, it is still an express condition that was waived.

The open-air amphitheater in the city park of a town has been utilized for concerts and other entertainment programs. Until this year, each of the groups performing in that city facility was allowed to make its own arrangements for sound equipment and sound technicians. After recurring complaints from occupants of residential buildings adjacent to the city park about intrusive noise from some performances held in the amphitheater, the town's city council passed an ordinance establishing city control over all sound amplification at all programs held there. The ordinance provided that the town's Department of Parks would be the sole provider in the amphitheater of sound amplification equipment and of the technicians to operate the equipment "to ensure a proper balance between the quality of the sound at such performances and respect for the privacy of nearby residential neighbors." Which of the following standards should a court use to determine the constitutionality on its face of this content neutral ordinance?

The ordinance is narrowly tailored to serve a substantial government interest, and does not unreasonably limit alternative avenues of expression The government may regulate the time, place, and manner of speech as long as that regulation: 1) is content neutral, 2) serves a substantial governmental interest, and 3) is narrowly tailored so that there are alternative avenues of expression open. This regulation is a time, place, and manner regulation because it affects only the volume of the expression. B, C, and D give the incorrect standards for this type of regulation.

The open-air amphitheater in the city park of a town has been utilized for concerts and other entertainment programs. Until this year, each of the groups performing in that city facility was allowed to make its own arrangements for sound equipment and sound technicians. After recurring complaints from occupants of residential buildings adjacent to the city park about intrusive noise from some performances held in the amphitheater, the town's city council passed an ordinance establishing city control over all sound amplification at all programs held there. The ordinance provided that the town's Department of Parks would be the sole provider in the amphitheater of sound amplification equipment and of the technicians to operate the equipment "to ensure a proper balance between the quality of the sound at such performances and respect for the privacy of nearby residential neighbors." Which of the following standards should a court use to determine the constitutionality on its face of this content neutral ordinance?

The ordinance is narrowly tailored to serve a substantial government interest, and does not unreasonably limit alternative avenues of expression. The government may regulate the time, place, and manner of speech as long as that regulation: 1) is content neutral, 2) serves a substantial governmental interest, and 3) is narrowly tailored so that there are alternative avenues of expression open. This regulation is a time, place, and manner regulation because it affects only the volume of the expression. B, C, and D give the incorrect standards for this type of regulation.

A seller entered into a written contract to sell a tract of land to an investor. The contract made no mention of the quality of title to be conveyed. The seller and the investor completed the sale, and the seller delivered a warranty deed to the investor. Soon thereafter, the value of the land increased dramatically. The investor entered into a written contract to sell the land to a buyer. The contract between the investor and the buyer expressly provided that the investor would convey a marketable title. The buyer's attorney discovered that the title to the land was not marketable, and had not been marketable when the original seller conveyed to the investor. The buyer refused to complete the sale. The investor sued the original seller for breach of contract, claiming damages from the seller's failure to convey marketable title, which resulted in the investor's loss of the sale to the subsequent buyer. Who is likely to prevail on this count?

The original seller, because her contract obligations as to title merged into the deed Although a marketable title will be implied in a contract for the sale of land, the doctrine of merger provides that one can no longer sue on title matters contained in the contract of sale after the deed is delivered and accepted. The investor's remedy, if there is one, would be based on the deed he received and not on the contract of sale.

Until 1954, a state required segregation in all public and private schools, but all public schools are now desegregated. Other state laws, enacted before 1954 and continuing to the present, provide for free distribution of the same textbooks on secular subjects to students in all public and private schools. In addition, the state accredits schools and certifies teachers. A private school offers elementary and secondary education in the state but denies admission to all non-Caucasians. Another private school offers religious instruction. Which of the following is the strongest argument in favor of the constitutionality of free distribution of textbooks to these students at the school offering religious instruction?

The purpose and effect of the free distribution of these textbooks is secular and does not entangle church and state The test to analyze whether a government action violates the Establishment Clause of the First Amendment is: (i) whether its purpose is secular; (ii) whether its primary effect advances or inhibits religion; and (iii) whether it fosters an excessive entanglement with religion. Therefore the strongest argument to support the constitutionality of the free distribution of textbooks to a religious institution is that its purpose and effect is secular and does not foster an excessive entanglement with religion.

A state statute provides that only citizens of the United States may be employed by that state. In an action brought in a federal court, a resident alien who was prevented from obtaining state employment as a garbage collector solely because of his alien status challenged the statute's constitutionality as applied to his circumstances. Which of the following statements concerning the burden of persuasion applicable to this suit is correct?

The state must demonstrate that the citizenship requirement is necessary to advance a compelling state interest. Alienage is a suspect class. Therefore, the state may only classify on the basis of alienage if the state can prove that the classification is narrowly tailored to promote a compelling state interest. State laws requiring United States citizenship for welfare benefits, civil service jobs, or a license to practice law will be struck down because there is no compelling state interest. However, there is an exception for alien participation in self-government. If a law discriminates against alien participation in the functioning of the state government, only rational basis review is applied. Certain privileges fall under this exception and have been reserved just for citizens: voting, serving on a jury, acting as a police officer, teacher, or probation officer, or any other position with a direct effect on the functioning of government. Garbage collector falls under a civil service job and is not part of the exception for participation in self-government. Therefore, strict scrutiny is used and not rational basis review.

A state has a statute providing that an unsuccessful candidate in a primary election for a party's nomination for elected public office may not become a candidate for the same office at the following general election by nominating petition or by write-in votes. A woman sought her party's nomination for governor in the May primary election. After losing in the primary, the woman filed nominating petitions containing the requisite number of signatures to become a candidate for the office of governor in the following general election. The chief elections officer of the state refused to certify the woman's petitions solely because of the above statute. The woman then filed suit in federal district court challenging the constitutionality of this state statute. As a matter of constitutional law, which of the following is the proper burden of persuasion in this suit?

The state must demonstrate that the statute is the least restrictive means of achieving a compelling state interest. Candidate qualifications need to be narrowly tailored to protect a compelling state interest to not violate equal protection or the First Amendment right of association, and the state has the burden to prove that its regulation meets strict scrutiny. Strict scrutiny in equal protection cases applies to both suspect classification and when a fundamental right is burdened by the classification the government has selected. The fundamental rights under equal protection that trigger strict scrutiny are the right to vote, the right to be a political candidate, the right to have access to the courts for certain kinds of proceedings, and the right to migrate interstate.

A chemical company designed and built a large tank on its premises for the purpose of storing highly toxic gas. The tank developed a sudden leak and escaping toxic gas drifted into the adjacent premises where a neighbor lived. The neighbor inhaled the gas and died as a result. In a suit brought by the neighbor's personal representative against the chemical company, which of the following states what must be established for the claim to prevail?

The toxic gas that escaped from the chemical company's premises was the cause of the neighbor's death. The operation of a storage facility that contains highly toxic chemicals is considered an abnormally dangerous activity. Ultra-hazardous or abnormally dangerous activities give rise to strict liability because the inherent danger or peculiar risk is unreasonably high when compared to its social utility, even in the absence of negligence and where all the proper precautions have been taken. Consequently, the neighbor's estate will prevail in a claim for damages against the chemical company as owner of the storage facility as long as it can be shown that the escaped gases caused the death; it is irrelevant whether the facility was defective or the chemical company was negligent in the operation of the property.

A park ranger filed suit against a scout, a rancher, and researcher, in state court, asserting claims under federal law. The scout was properly served with the summons and complaint on February 1, 2014. The rancher was properly served with the summons and complaint on March 5, 2014. The researcher was properly served with the summons and complaint on April 7, 2014. On April 17, the researcher filed a timely notice of removal with the proper local federal district court. The scout then immediately thereafter filed his own notice of removal. Two weeks after April 17, the rancher then filed with the court a letter stating his consent to the removal of the case. The park ranger then moved for remand to state court. Should the federal district court grant the park ranger's motion to remand?

There were no defects in the removal procedure The federal district court should deny the park ranger's motion to remand because there were no defects in the removal process, and the court has subject-matter jurisdiction over the case. The case arises under federal law, so the court has federal question jurisdiction. According to 28 U.S.C. § 1446, notice of removal is valid within 30 days of service of the complaint on the defendant. If multiple defendants are served at different times and a later served defendant initiates removal, the earlier served defendants may join in the removal, even if their original 30-day window has closed. In this question, all defendants consented to removal within 30 days of the notice of removal initiated by the researcher.

A lumberjack properly brought a single-count complaint alleging medical malpractice against a doctor in the federal district court of State A under diversity jurisdiction. The jury found for the lumberjack and awarded him $2.5 million in damages. Federal law permits a judge to set aside a jury's verdict and order a new trial if the jury's award is so high that it "shocks the conscience." Whereas, State A's tort reform statute authorizes a judge to order a new trial only upon a finding of "excessive damages." May the federal district court order a new trial?

Yes, after a finding that the jury awarded excessive damages Because this lawsuit is based on diversity jurisdiction, the question of which law to apply is an Erie question. Further, in Gasperini v. Center, 518 U.S. 415 (1996), the Supreme Court has required federal trial courts to apply a state standard when considering a motion for a new trial based on excessiveness of the verdict. The Federal Rules of Civil Procedure do not explicitly enumerate the "shocks the conscience" rule, so the appropriate analysis is Erie's constitutional test. This test asks whether the potentially applicable state law is substantive or procedural. Under the Erie Doctrine, a federal court is required to apply the substantive law of the state in which it is sitting, including that state's conflict of law rules. This question refers to a matter of substantive law. Therefore, the court must follow State A's tort reform statute and apply the state standard of "excessive damages."

Congress enacted a statute providing grants of federal funds for the restoration and preservation of courthouses that were built before 1900 and are still in use. The statute contains an inseverable condition requiring that any courthouse restored with the aid of such a grant must be equipped with ramps and other facilities necessary to accommodate physically handicapped people. A law of a particular state requires public buildings in the state to have ramps and other facilities for handicapped people. It exempts from those requirements any building that is more than 70 years old if the State Board of Architects finds that the installation of such facilities would destroy the architectural integrity of the building. The county courthouse in the state was built in 1895 and is still in use. It does not contain ramps or other special facilities for handicapped people. The State Board of Architects has determined that the installation of those facilities would destroy the architectural integrity of the building. The county board applies for a federal grant to restore and preserve that county's courthouse. If the county board restores the courthouse with the aid of a federal restoration and preservation grant, is the board bound to install ramps and other facilities for handicapped people in that building?

Yes, because Congress may impose reasonable conditions related to the public welfare on grants of federal funds to public bodies when the public bodies are free to accept or reject the grants. Congress can condition the grant of federal funds to public bodies on compliance with measures related to the public welfare as long as the public body is free to accept or reject the grants.

An investor from State A filed an action against his State B stockbroker in federal court in State A. The summons and complaint were served at the stockbroker's office in State B, where the process server handed the documents to the stockbroker's administrative assistant. The stockbroker has answered the complaint, asserting the defense of improper service of process. Assume that both states' requirements for service of process are identical to the requirements of the Federal Rules of Civil Procedure. Is the court likely to dismiss the action for improper service of process?

Yes, because an individual defendant may not be served by delivering process to a third party found at the defendant's place of employment Federal Rule of Civil Procedure 4(e)(2) provides that an individual defendant may be served by delivering a copy of the summons and complaint to an agent authorized by appointment or by law to receive service of process on behalf of the defendant. No facts suggest that the administrative assistant was a designated agent of the stockbroker, and the Rules provide no general authority to serve process on third parties at a defendant's place of employment. A designated agent may be an officer, a managing or general agent, or someone specifically labeled an authorized agent. In this fact pattern, none of these apply to the administrative assistant.

On July 15, in a writing signed by both parties, a fixture company agreed to deliver to a druggist on August 15 five storage cabinets from inventory for a total price of $5,000 to be paid on delivery. On August 1, the two parties orally agreed to postpone the delivery date to August 20. On August 20, the fixture company tendered the cabinets to the druggist, who refused to accept or pay for them on the ground that they were not tendered on August 15, even though they otherwise met the contract specifications. Assuming that all appropriate defenses are seasonably raised, will the fixture company succeed in an action against the druggist for breach of contract?

Yes, because the August 1 agreement operated as a waiver of the August 15 delivery term Waiver is an excuse for the nonoccurrence or delay of a condition to fulfill a duty. Under the UCC, consideration is not required for a valid waiver. When the fixture company and the druggist agreed to postpone their delivery date under the contract from August 15 to August 20, in effect the druggist waived a non-material term of the agreement and the waiver was enforceable even without consideration. The druggist, therefore, was not justified in refusing to accept or to pay for the cabinets.

A teenager and two of his friends were members of a teenage street gang. While they were returning from a dance late one evening, their car collided with a car driven by an elderly woman. After an argument, the teenager attacked the elderly woman with his fists and beat her to death. The teenager's two friends watched, and when they saw the woman fall to the ground they urged the teenager to flee. The teenager was eventually apprehended and tried for manslaughter, but the jury could not decide on a verdict. If the teenager's companions are subsequently tried as accomplices to manslaughter, they should be

acquitted, because they did not assist or encourage the teenager to commit the crime. Don't be lured in by the fact that the friends were members of a street gang or that the victim was a helpless elderly lady. The Examiners will try to sway your emotions and trick you into a wrong choice. B is the best answer because it addresses the fact that the friends stood there and did not encourage or help the teenager in his actions and so did not aid in the commission of manslaughter.

A plaintiff entered a drug store to make some purchases. As he was searching the aisles for various items, he noticed a display card containing automatic pencils. The display card was on a high shelf behind a cashier's counter. The plaintiff saw a sign on the counter that read, "No Admittance, Employees Only." Seeing no clerks in the vicinity to help him, the plaintiff went behind the counter to get a pencil. A clerk then appeared behind the counter and asked whether she could help him. He said he just wanted a pencil and that he could reach the display card himself. The clerk said nothing further. While reaching for the display card, the plaintiff stepped sideways into an open shaft and fell to the basement, ten feet below. The clerk knew of the presence of the open shaft, and had reason to believe that the plaintiff had not noticed it when stepping behind the counter. The plaintiff sued the drug store to recover damages for the injuries he sustained in the fall. The jurisdiction has adopted a rule of pure comparative negligence, and it follows traditional common-law rules governing the duties of a land possessor. Will the plaintiff recover a judgment against the drug store?

Yes, because the clerk had reason to believe that the plaintiff was unaware of the open shaft. A business has a duty to make a reasonable inspection with respect to invitees and a duty to warn of or repair known hazards on the premises. The duty applies to defects that are (i) known to the business, (ii) dangerous, and (iii) non-obvious to the invitee. Liability may be abated if the invitee becomes a trespasser by entering an area that was restricted and off-limits to visitors. However, an owner still has the duty to warn a discovered trespasser of a peril known to the owner. Here, the plaintiff was originally an invitee but became a trespasser when he went beyond the scope of the drug store's invitation. However, once the clerk discovered the plaintiff and allowed the entry, the clerk had at least a duty to warn the plaintiff of the open shaft.

A property owner and a landscape architect signed a detailed writing in which the landscape architect agreed to landscape the owner's residential property in accordance with a design prepared by the landscape architect and incorporated in the writing. The owner agreed to pay $10,000 for the work upon its completion. The owner's spouse was not a party to the agreement and had no ownership interest in the premises. Shortly before the agreement was signed, the owner and landscape architect orally agreed that the writing would not become binding on either party unless the owner's spouse should approve the landscaping design. If the owner's spouse disapproves of the design and the owner refuses to allow the landscape architect to proceed with the work, is evidence of the oral agreement admissible in the landscape architect's action against the owner for breach of contract?

Yes, because the evidence shows that the writing was intended to take effect only if the approval occured The parol evidence rule bars extrinsic evidence of a prior agreement either where the prior agreement contradicts the terms of a final written agreement or where the prior agreement purports to add to a completely integrated agreement (i.e., one that is intended by the parties to be both the final and exclusive manifestation of the parties' understanding). An exception to the parol evidence rule applies where the extrinsic evidence is offered to prove the existence of a condition precedent to the written agreement. Therefore, evidence of the owner and landscape architect's oral agreement is admissible because it demonstrated the existence of a condition precedent to the writing's taking effect.

The aged mother of a sister and brother, both adults, wished to employ a live-in companion so that she might continue to live in her own home. Their mother, however, had only enough income to pay one-half of the companion's $2,000 monthly salary. Learning of their mother's plight, the siblings agreed with each other in a signed writing that on the last day of January and each succeeding month during their mother's lifetime, each would give their mother $500. Their mother then hired the companion. The siblings made the agreed payments in January, February, and March. In April, however, the brother refused to make any payment and notified his sister and mother that he would make no further payments. Will their mother succeed in an action for $500 brought against the brother after April 30?

Yes, because the mother is an intended beneficiary of a contract between the siblings The mother was not a party to the contract between the sister and the brother, so the only way for her to have contractual rights is if she was the intended beneficiary of the contract. For the mother to be an intended beneficiary, she must be the person to whom performance is to be given. In this case, several factors indicate that the mother is an intended beneficiary: the mother is named in the contract, the mother directly receives the payments, and the mother has a close familial relationship with the parties. Therefore, the mother is an intended beneficiary and will prevail in a suit against the brother for the $500 payment.

A buyer mailed a signed order to a seller that read: "Please ship us 10,000 widgets at your current price." The seller received the order on January 7 and that same day mailed to the buyer a properly stamped, addressed, and signed letter stating that the order was accepted at the seller's current price of $10 per widget. On January 8, before receipt of the seller's letter, the buyer telephoned the seller and said, "I hereby revoke my order." The seller protested to no avail. The buyer received the seller's letter on January 9. Because of the buyer's January 8 telephone message, the seller never shipped the goods. Under the relevant and prevailing rules, is there a contract between the buyer and the seller as of January 10?

Yes, because the order was an offer that seller effectively accepted before the buyer attempted to revoke it Unless the offer is an option, the offeror may revoke an offer at any time before the offer has been accepted. Under the mailbox rule, unless the offer specifies otherwise, notice of acceptance is effective upon dispatch in the mail if a mailed acceptance is invited by the terms of the offer. Because the buyer's order was mailed, the seller's properly stamped and addressed letter was invited by the terms of the offer and was therefore effective upon its dispatch on January 7. Because the buyer did not attempt to revoke the offer until January 8, after the seller had already dispatched their acceptance on January 7, the seller created a contract that was effective on January 7. The buyer's revocation after the acceptance is not enough to terminate the contract.

A plaintiff, who was 20 years old, purchased a new, high-powered sports car that was marketed with an intended and recognized appeal to youthful drivers. The car was designed with the capability to attain speeds in excess of 100 miles per hour. It was equipped with tires designed and tested only for a maximum safe speed of 85 miles per hour. The owner's manual that came with the car stated that "continuous driving over 90 miles per hour requires high-speed-capability tires," but the manual did not describe the speed capability of the tires sold with the car. The plaintiff took her new car out for a spin on a straight, smooth country road where the posted speed limit was 55 miles per hour. Intending to test the car's power, she drove for a considerable distance at over 100 miles per hour. While she was doing so, the tread separated from the left rear tire, causing the car to leave the road and hit a tree. The plaintiff sustained severe injuries. The plaintiff has brought a strict product liability action in tort against the manufacturer of the car. You should assume that pure comparative fault principles apply to this case. Will the plaintiff prevail?

Yes, because the statement in the manual concerning the tires did not adequately warn of the danger of high-speed driving on the tires mounted on the car. The car manufacturer created a high-powered sports car and then failed to equip it with high-speed capability tires. This would make the car unreasonably dangerous and strict liability may be applied if, as in this case, the manufacturer failed to give a proper warning as to the type of tires needed for the car to be driven at its higher speeds. The duty to warn will create strict liability despite the plaintiff's prolonged use of the car at a high speed because its use at that high speed was a foreseeable use, given its design and marketing. While the car was not defective, the failure to give proper directions and specific warning was.

On June 1, a manufacturer of men's neckties received the following order from a retailer: "Ship 500 two-inch ties, assorted stripes, your catalog No. V34. Delivery by July 1." On June 1, the manufacturer shipped 500 three-inch ties that arrived at the retailer's place of business on June 3. The retailer immediately telegraphed the manufacturer: "Reject your shipment. Order was for two-inch ties." The retailer, however, did not ship the ties back to the manufacturer. The manufacturer replied by telegram: "Will deliver proper ties before July 1." The retailer received this telegram on June 4, but did not reply to it. On June 30, the manufacturer tendered 500 two-inch ties in assorted stripes, designated in his catalog as item No. V34, but the retailer refused to accept them. Did the retailer properly reject the ties delivered on June 3?

Yes, because the ties were nonconforming goods The retailer satisfied its duty to promptly notify the manufacturer that the goods were rejected as nonconforming. According to the perfect tender rule, if goods or their delivery fail to conform to the contract in any way, https://app.adaptibar.com/student/assignments/report/1860 14/30 5/2/2019 AdaptiBar Online MBE Simulator & Prep | AdaptiBar the buyer may reject all, accept all, or accept any commercial units and reject the rest. In this question, the retailer decided to reject the non-conforming goods, and the manufacturer is not able to cure.

An accountant from State A sues a biologist from State A in federal court in State A. The accountant does not state a claim based on federal law in her complaint. However, the accountant anticipates the biologist will use a federal law in his defense. Should the judge dismiss the accountant's complaint?

Yes; the judge should dismiss the accountant's complaint because the complaint does not contain a federal question The federal question must appear as part of the accountant's cause of action as set out in a well-pleaded complaint. The fact that the biologist's defense may be based in federal law is irrelevant and not sufficient to justify federal question jurisdiction over the accountant's claim. The court may not look to a defense asserted by the biologist to determine whether the accountant's complaint raises a federal question.

A plaintiff suffered a severe loss when his manufacturing plant, located in a shallow ravine, was flooded during a sustained rainfall. The flooding occurred because the city had failed to maintain its storm drain, which was located on city land above the plaintiff's premises, and because a railroad had failed to maintain its storm drain, which was located on railroad land below the plaintiff's premises. The flooding would not have occurred if either one of the two storm drains had been maintained properly. The plaintiff sued the railroad to recover compensation for his loss. The evidence in the case established that the failures of the two drains were caused by the respective negligence of the city and the railroad. There is no special rule insulating the city from liability. In his action against the railroad, the plaintiff should recover

all of his loss, because but for the railroad's negligence none of the flooding would have occurred. Under the facts, the plaintiff's damages resulted from the negligence of two independent entities, neither of which alone would have been sufficient to flood the plaintiff's manufacturing plant. The railroad's failure to keep its storm drain properly maintained was a material element and, therefore, a substantial factor in bringing about the flooding. The plaintiff does not need to join the city in the suit and is not required to prove damages against the city. The railroad can argue the city's negligence in an effort to apportion or limit damages, implead the city as a third-party defendant, or pursue an action for contribution from the city after the railroad's judgment. The city's absence from the plaintiff's action will not affect the railroad's liability as a jointly liable concurrent tortfeasor, for the entire amount of the plaintiff's awarded damages.

A pedestrian started north across the street in a clearly marked north-south crosswalk with the green traffic light in her favor. The pedestrian was in a hurry, and so before reaching the north curb on the street, she cut to her left diagonally across the street to the east-west crosswalk and started across it. Just after reaching the east-west crosswalk, the traffic light turned green in her favor. She proceeded about five steps further across the street to the west in the crosswalk when she was struck by a car approaching from her right that she thought would stop, but did not. The car was driven by a driver, 81 years of age, who failed to stop his car after seeing that the traffic light was red against him. The pedestrian had a bone disease, resulting in very brittle bones, that is prevalent in only 0.02 percent of the population. As a result of the impact, the pedestrian suffered a broken leg and the destruction of her family heirloom, a Picasso original painting that she was taking to her bank for safekeeping. The painting had been purchased by the pedestrian's grandmother for $750 but was valued at $500,000 at the time of the accident. The pedestrian has filed suit against the driver. The driver's attorney has alleged that the pedestrian violated a state statute requiring that pedestrians stay in crosswalks, and that if the pedestrian had not violated the statute, she would have had to walk 25 feet more to reach the impact point and therefore would not have been at a place where she could have been hit by the driver. The pedestrian's attorney ascertains that there is a statute as alleged by the driver, that his measurements are correct, that there is a state statute requiring observance of traffic lights, and that the driver's license expired two years prior to the collision. The pedestrian's violation of the crosswalk statute should not be considered by the jury because

as a matter of law the injury to the pedestrian was not the result of a risk the statute was designed to protect against. Safety statutes are used to establish duty and breach only if the violated statute was designed to protect the particular class of foreseeable victim that the plaintiff belonged to, and the harm the plaintiff suffered was the type that the statute was designed to protect. Here, the pedestrian was hit by the driver when she was already in the other crosswalk, so she was not in the class of people that the statute was designed to protect, namely, pedestrians crossing streets outside of crosswalks. This answer choice appropriately acknowledges the test and distinction and is the correct response.

After waiting until all the customers had left, a man entered a small grocery store just before closing time. He went up to the lone clerk in the store and said, "Hand over all the money in the cash register or you will get hurt." The clerk fainted and struck his head on the edge of the counter. As the man went behind the counter to open the cash register, two customers entered the store. The man ran out before he was able to open the register drawer. On this evidence the man could be convicted of

attempted robbery. Robbery is the taking, by force or threat of force, the personal property of another with the intent to permanently deprive the owner of the property. In the question, the man was unable to take possession of any personal property of another and thus, did not commit the crime of robbery. To be convicted of attempted robbery, a person must have the intent to commit a robbery and take a substantial step toward the commission of the offense. The man did, by threatening the clerk and going behind the counter, take substantial steps toward the commission of the offense of robbery, and did have the intent to commit the robbery, so he should be convicted of attempted robbery.

A generally applicable state statute requires an autopsy by the county coroner in all cases of death that are not obviously of natural causes. The purpose of this law is to ensure the discovery and prosecution of all illegal activity resulting in death. In the 50 years since its enactment, the statute has been consistently enforced. A husband and wife are sincere practicing members of a religion that maintains it is essential for a deceased person's body to be buried promptly and without any invasive procedures, including an autopsy. When the couple's son died of mysterious causes and an autopsy was scheduled, the couple filed an action in state court challenging the constitutionality of the state statute, and seeking an injunction prohibiting the county coroner from performing an autopsy on their son's body. In this action, the couple claimed only that the application of this statute in the circumstances of their son's death would violate their right to the free exercise of religion as guaranteed by the First and Fourteenth Amendments. Assume that no federal statutes are applicable. As applied to the couple's case, the court should rule that the state's autopsy statute is

constitutional, because it is a generally applicable statute and is rationally related to a legitimate state purpose. When a neutral law of general applicability impacts a religious practice, the law is subject to rational basis review. There is no inquiry into the extent of the impact or the sincerity of the religious beliefs. The law must merely be rationally related to a conceivable legitimate state interest.

A clerk lived on the second floor of a small convenience store/gas station that he owned. One night he refused to sell the defendant a six-pack of beer after hours, saying he could not violate the state laws. The defendant became enraged and deliberately drove his car into one of the gasoline pumps, severing it from its base. There was an ensuing explosion causing a ball of fire to go from the underground gasoline tank into the building. As a result, the building burned to the ground and the clerk was killed. In a common-law jurisdiction, if the defendant is charged with murder and arson, he should be

convicted of both offenses. At common law, murder is defined as the unlawful killing of another human being with malice aforethought. Malice can be shown by having an intent to kill, or by having an intent to inflict great bodily harm. Malice can also be implied in actions that demonstrate gross recklessness toward human life. In addition, felony murder is the killing of someone during the commission of a violent felony. Arson is defined at common law as the malicious burning of the dwelling of another. In this case, the defendant maliciously burned the clerk's dwelling and, during the commission of that arson, caused the clerk's death. Additionally, the defendant acted with malice aforethought when he caused the clerk's death. Deliberately driving a car into a gasoline pump demonstrates a reckless indifference to a high risk to human life; this is sufficient for a finding of malice aforethought for the murder charge. The defendant could be convicted of either murder with malice aforethought or felony murder with arson as the underlying felony. In addition, the defendant's driving his vehicle into a gasoline pump also had the foreseeable consequences of igniting the gasoline, creating a ball of fire, and burning the building the clerk was in. Therefore, the defendant is guilty of both arson and murder.

A man was a suspect in a homicide committed during a robbery of a liquor store. A barber was a friend of the suspect. The police telephoned the barber and asked if he would help locate the suspect. The barber agreed and met the police officers at headquarters later that night. After a discussion during which police asked questions about the suspect and the homicide, the barber said he wanted to get something "off his chest" and advised the officers that he was in on the robbery but that the suspect had shot the owner of the store without his permission or prior knowledge. The officers then for the first time gave the barber his Miranda warnings. The barber was indicted for felony murder. He moved to prevent the introduction of his statement into evidence. His motion should be

denied, because his statement was freely and voluntarily given, and he was not entitled to Miranda warnings. The barber's statements were made voluntarily and without police coercion. The officers were questioning the barber in an attempt to locate information and the whereabouts of the suspect, not in an attempt to obtain a confession from the barber. The barber was not the suspect, never placed under arrest, and not subject to custodial interrogation, so the police officers were not constitutionally required to give him his Miranda warnings. The barber's motion to suppress his statement should be denied.

A defendant left her car parked on the side of a hill. Two minutes later, the car rolled down the hill and struck and injured the plaintiff. In the plaintiff's negligence action against the defendant, the plaintiff introduced into evidence the facts stated above, which are undisputed. The defendant testified that, when she parked her car, she turned the front wheels into the curb and put on her emergency brakes, which were in good working order. She also introduced evidence that, in the weeks before this incident, juveniles had been tampering with cars in the neighborhood. The jury returned a verdict in favor of the defendant, and the plaintiff properly moved for a judgment notwithstanding the verdict. The plaintiff's motion should be

denied, because, given the defendant's evidence, the jury was not required to draw an inference of negligence from the circumstances of the accident. A motion for judgment notwithstanding the verdict (also called a renewed motion for judgment as a matter of law) requires a finding by the judge that the verdict could not have been reached by a reasonable jury. The plaintiff attempted to prove her claim using the doctrine of res ipsa loquitur. The doctrine of res ipsa loquitur is generally applied in situations where negligence clearly occurred and (1) the defendant had exclusive control of the instrumentality during the relevant time, and (2) the plaintiff shows that he was not responsible for the injury. The plaintiff might have prevailed under res ipsa loquitur if it wasn't for the defendant's showing that she may not have been in exclusive control of the instrumentality. Because of the evidence to the contrary, the jury need not have inferred negligence from the circumstances of the accident. A verdict that could have been reached by a reasonable jury will not be overturned. In this situation, there was enough evidence in favor of the defendant that the jury's decision was not unreasonable. C is correct. It states the appropriate standard for a motion for judgment notwithstanding the verdict.

The legislature of a particular state enacted a statute requiring that all law enforcement officers in that state be citizens of the United States. An alien, lawfully admitted to permanent residency five years before the enactment of this statute, sought employment as a forensic pathologist in the state coroner's office. He was denied such a job solely because he was not a citizen. The alien thereupon brought suit in federal district court against appropriate state officials seeking to invalidate this citizenship requirement on federal constitutional grounds. The strongest ground upon which to attack this citizenship requirement is that it

denies an alien the equal protection of the laws guaranteed by the Fourteenth Amendment The Equal Protection Clause requires that classifications based on alienage be narrowly tailored to promote a compelling state interest. Because that is such a difficult test to satisfy, this is the strongest argument.

A real estate company acquired a large tract of land upon which the company developed a mobile home subdivision. The tract was divided into 60 lots, appropriate utilities were installed, and a plat of the entire tract, including a Declaration of Restrictions, was properly drawn and recorded. The Declaration of Restriction included the following: "3. Ownership and/or occupancy are restricted to persons 21 years of age or over, one family per lot." As the separate lots were sold, the deed to each lot included the following provision: "As shown on recorded plat [properly identified by page and plat book reference] and subject to the restrictions therein contained." One of the lots was purchased by a man, who now resides in a mobile home on the lot together with his wife and two children, aged 11 and 13. Other lot owners in the subdivision brought an action against the man to enjoin further occupancy by the children under 21 years of age. If judgment is entered for the man, the issue that most likely will determine the case will be whether

enforcement of the restriction is considered a violation of the Equal Protection Clause of the Fourteenth Amendment of the United States Constitution. While normally equal protection does not apply to private conduct, if the government affirmatively facilitates unconstitutional activity, the conduct is subject to review. Therefore, if the lot owners are seeking to enforce an age-based restrictive covenant via the courts, the entanglement exception applies and an equal protection review of the classification is triggered.

The following events took place in a state that does not recognize common-law marriage. The state does recognize the common-law estate of tenancy by the entirety and has no statute on the subject. A man and woman, who were never formally married, lived together over a seven-year period. During this time the woman identified herself using the man's last name with the knowledge and consent of the man. The couple maintained several charge accounts at retail stores under the husband's last name, and they filed joint income tax returns using his last name for both of them. Within this period the man decided to buy a home. The deed was in proper form and identified the grantees as "Mr. and Mrs. [last name], and their heirs and assigns forever as tenants by the entirety." The man made a down payment of $10,000 and gave a note and mortgage for the unpaid balance. The couple signed the note and mortgage as husband and wife. The man made monthly payments as they became due until he and the woman had a disagreement and he abandoned her and the house. The woman then made the payments for three months. She then brought an action against the man for partition of the land in question. The prayer for partition should be

granted, because the estate created by the deed was not a tenancy by the entirety This question requires you to be able to identify a tenancy by the entirety. Similar to a joint tenancy, a tenancy by the entirety requires the presence of several unities (time, title, interest, possession and person). However, unlike a joint tenancy, this type of tenancy requires that the tenants be married. A tenancy by the entirety features the right of survivorship, and the other distinguishing feature of a tenancy by the entirety is the limit on the ways that it can be severed. A tenancy by the entirety may only be severed: (i) by divorce; (ii) by the death of one spouse; (iii) by a creditor of both spouses; or (iv) by a mutual agreement between the husband and wife. In this case, the man and woman are not husband and wife, and are thereby prevented from entering into a marital estate. Further, the tenancy may be severed by partition because the man and woman did not enter into tenancy by the entirety.

A professional poker player had cheated a man in a card game. Angered, the man set out for the poker player's house with the intention of shooting him. Just as he was about to set foot on the poker player's property, the man was arrested by a police officer who noticed that the man was carrying a revolver. A statute in the jurisdiction makes it a crime to "enter the property of another with the intent to commit any crime of violence thereon." If charged with attempting to violate the statute, the man should be found

guilty, because he was close enough to entering the property and he had the necessary state of mind To be found guilty of attempt to commit a crime, one must have the intent that the crime be committed and take a substantial step towards its commission. In this question, the statutory crime is to enter the property of another with the intent to commit any crime of violence thereon. The question indicates that the man clearly had the intent to shoot the poker player, and the man clearly had the intent to enter the poker player's property to do so. Obtaining the weapon and going over to the poker player's house are substantial steps towards the commission of the statutory offense. The man intended to enter the poker player's property in order to cause violence and took substantial steps toward the commission of that offense; thus, he should be convicted of attempt to commit the offense.

The defendant visited a fellow college student in the student's dormitory room. They drank some beer. The student produced a box containing marijuana cigarettes and asked if the defendant wanted one. The defendant, afraid of being caught, declined and urged the student to get rid of the marijuana. The student refused. Shortly thereafter, both went out to get more beer, leaving the door to the student's room standing open. Making an excuse about having dropped his pen, the defendant went back into the student's room. Still apprehensive about their being caught with the marijuana cigarettes, he took the cigarettes and flushed them down the toilet. He was sure the student was too drunk to notice that the cigarettes were missing. The defendant is charged with larceny and burglary (defined in the jurisdiction as breaking and entering the dwelling of another with intent to commit any felony or theft). He should be found guilty of

larceny only. The defendant took the property of another, the student's marijuana, without the owner's consent and with the intent to permanently deprive the owner of the property. It is irrelevant that the owner's possession of the property was likely illegal. Thus, the defendant should be found guilty of larceny. Here, burglary is defined as breaking and entering the dwelling of another with intent to commit any felony or theft. The defendant is not guilty of burglary because the door was open, so the defendant did not break into the dormitory room. Thus, A, C, and D are incorrect.

A homeowner owns a house in a city. On the lawn in front of his home and within five feet of the public sidewalk there was a large tree. The roots of the tree caused the sidewalk to buckle severely and become dangerous. An ordinance of the city requires adjacent landowners to keep sidewalks in a safe condition. The homeowner engaged a contractor to repair the sidewalk, leaving it to the contractor to decide how the repair should be made. The contractor dug up the sidewalk, cut back the roots of the tree, and laid a new sidewalk. Two days after the homeowner had paid the contractor the agreed price of the repair, the tree fell over onto the street and damaged a parked car belonging to a driver. The driver has asserted claims against the homeowner and the contractor, and both defendants admit that cutting the roots caused the tree to fall. The homeowner also admitted that he was aware of the dangerous manner in which the contractor was performing the repairs. The theory on which the driver is most likely to prevail against the homeowner is that the homeowner is

liable for the contractor's negligence because the homeowner knew the contractor was engaged in a dangerous activity. Ordinarily, a landowner would not be liable for the acts of an independent contractor, so long as the contractor was not negligently hired. Two broad exceptions exist: (i) the independent contractor is engaged in inherently dangerous activities (e.g., excavating next to a public sidewalk, blasting); or (ii) the duty is non-delegable (duty to keep premises safe). In this case, because the contractor was engaged in an inherently dangerous activity, and because the homeowner was aware of the contractor's work, the homeowner will be liable for the damage.

At a party, the defendant and the victim agreed to play a game they called "spin the barrel." The victim took an unloaded revolver, placed one bullet in the barrel, and spun the barrel. The victim then pointed the gun at the defendant's head and pulled the trigger once. The gun did not fire. The defendant then took the gun, pointed it at the victim, spun the barrel, and pulled the trigger once. The gun fired, and the victim fell over dead. A statute in the jurisdiction defines murder in the first degree as an intentional and premeditated killing or one occurring during the commission of a common-law felony, and murder in the second degree as all other murder at common law. Manslaughter is defined as a killing in the heat of passion upon an adequate legal provocation or a killing caused by gross negligence. The most serious crime for which the defendant can properly be convicted is

murder in the second degree, because the defendant's act posed a great threat of serious bodily harm. The defendant's pointing a loaded weapon at the victim and pulling the trigger posed a great threat of serious bodily injury to the victim, and, because it resulted in the victim's death, the defendant should be convicted of murder in the second degree. Murder in the second degree is murder with malice aforethought, done without premeditation and deliberation. The defendant's actions were done with reckless indifference to an unjustifiably high risk to human life, and will support a finding of malice aforethought

While the defendant was in jail on a pending charge, his landlord called the police because rent had not been paid and because he detected a disagreeable odor coming from the defendant's apartment into the hallways. The police officer who responded to the call knew that the defendant was in jail. He recognized the stench coming from the defendant's apartment as that of decomposing flesh and, without waiting to obtain a warrant and using the landlord's passkey, entered the apartment with the landlord's consent. The lease to these premises gave the landlord a right of entry, at any reasonable hour, for the purpose of making repairs. The police officer found a large trunk in the bedroom which seemed to be the source of the odor. Upon breaking it open, he found the remains of the defendant's former mistress. The landlord's consent to the police officer's search of the defendant's apartment is

not a waiver of the defendant's Fourth Amendment rights, because the landlord had neither actual nor apparent authority to permit the entry. The landlord had no actual or apparent authority to permit the entry because landlords generally do not have such authority over the premises of their lessees, and the provision in the lease only allowed limited rights of entry to make repairs, not a general authority to control the premises. In fact, that the lease had to contain a provision to allow the landlord such a limited right of entry is an indication that the premises was controlled by the lessee alone. Without authority, the landlord could not waive the defendant's Fourth Amendment rights.

A man, his brother, and his cousin are charged in a common law jurisdiction with conspiracy to commit larceny. The state introduced evidence that they agreed to go to a neighbor's house to take stock certificates from a safe in the neighbor's bedroom, that they went to the house, and that they were arrested as they entered the neighbor's bedroom. The man testified that he thought the stock certificates belonged to the cousin, that the neighbor was improperly keeping them from the cousin, and that he went along to aid in retrieving the cousin's property. The brother testified that he suspected the man and the cousin of being thieves and joined up with them in order to catch them. He also testified that he made an anonymous telephone call to the police alerting them to the crime and that the call caused the police to be waiting for them when they walked into the neighbor's bedroom. The cousin did not testify. If the jury believes the brother, it should find him

not guilty, because he did not intend to steal. The charge of conspiracy requires that two parties intentionally enter an agreement to commit a crime with the intent to achieve the objective of the agreement. If the jury believes that the brother was attempting to catch two thieves, then the brother did not have the requisite intent to achieve the objective of the agreement (larceny). Therefore, he should be found not guilty of conspiracy to commit larceny.

A construction company was engaged in blasting operations to clear the way for a new road. The company had erected adequate barriers and posted adequate warning signs in the vicinity of the blasting. Although the plaintiff read and understood the signs, he entered the area to walk his dog. As a result of the blasting, the plaintiff was hit by a piece of rock and sustained head injuries. The jurisdiction follows the traditional common-law rules governing the defenses of contributory negligence, assumption of risk, and last clear chance. In an action by the plaintiff against the construction company to recover damages for his injuries, the plaintiff will

not prevail, because the plaintiff understood the signs and disregarded the warnings. Normally, contributory negligence is not a valid defense in a strict liability action. However, in ultra-hazardous activity cases, contributory negligence is a defense if the plaintiff knew of the danger and his unreasonable conduct was the reason the activity caused injury. In such a case, the plaintiff's conduct is deemed knowing contributory negligence, and will be treated as an assumption of risk. Here, the plaintiff read and understood the signs but unreasonably entered the area anyway. The plaintiff's decision to enter the dangerous area after being properly warned was the reason the blasting caused an injury, raising his actions to knowing contributory negligence. The plaintiff's assumption of risk will relieve the construction company of liability for his injuries.

On May 1 a lot owner telegraphed a buyer, "Will sell you any or all of the lots in the Grover subdivision at $5,000 each. Details will follow in a letter." The letter contained all the necessary details concerning terms of payment, insurance, mortgages, etc., and provided, "This offer remains open until June 1." On May 2, after he had received the telegram but before he had received the letter, the buyer telegraphed the owner, "Accept your offer with respect to lot 101." Both parties knew that there were fifty lots in the Grover subdivision and that they were numbered 101 through 150. Assume that on May 5 the owner telephoned the buyer to explain that he had sold lots 102 through 150 to someone else on May 4 and that the buyer thereafter telegraphed the owner, "Will take the rest of the lots." Assume further that there is no controlling statute. In an action by the buyer against the owner for breach of contract, the buyer probably will

not succeed, because the buyer's power of acceptance was terminated by an effective revocation An offeror is free to revoke his offer any time before it is accepted, except for limited situations where the offer is irrevocable. Revocation by the offeror becomes effective upon receipt by the offeree. Further, an offer can be revoked indirectly when the offeror acts inconsistently with his intent to enter into the contract and the offeree finds out. Here, the owner's telephone call on May 5 informing the buyer that the remaining lots had been sold was a manifestation of the owner's intention not to enter into a contract with the buyer for the remaining lots. Here, because the owner's acts were inconsistent with the intent of the offer and the buyer had knowledge after their phone call, the offer was revoked by the phone call before the buyer's telegram accepting for the rest of the lots.

A builder purchased a large tract of land intending to construct residential housing on it. The builder hired a contractor to build a large in-ground swimming pool on the tract. The contract provided that the contractor would carry out blasting operations that were necessary to create an excavation large enough for the pool. The blasting caused cracks to form in the walls of the plaintiff's home in a nearby residential neighborhood. In the plaintiff's action for damages against the builder, the plaintiff should

prevail, because the blasting that the contractor was hired to perform damaged the plaintiff's home. In general, an independent contractor is liable for his own torts. An exception occurs when the contractor is carrying out an inherently dangerous activity on the property of his employer, causing an injury or trespass.

A plaintiff and a man were passengers sitting in adjoining seats on a flight on an airline. There were many empty seats on the aircraft. During the flight, a flight attendant served the man nine drinks. As the man became more and more obviously intoxicated and attempted to engage the plaintiff in a conversation, the plaintiff chose to ignore the man instead of changing seats. This angered the man, who suddenly struck the plaintiff in the face, giving him a black eye. The flight attendant had witnessed the man becoming violent but chose to not get involved. If the plaintiff asserts a claim for damages against the airline based on negligence, the plaintiff will

recover, because the flight attendants should have perceived the man's condition and acted to protect the plaintiff before the blow was struck. The airline has a duty to take reasonable steps to make the conditions on its aircraft flights reasonably safe, to conduct active operations with reasonable care for the presence of its customers, and, in appropriate circumstances, to use reasonable care to protect its customers against the foreseeable harmful/criminal acts of third persons or animals. If the flight attendant served the man nine drinks, the issue becomes one of whether the attendant should have paid attention to the man's state of intoxication and his general behavior, because unruly or dangerous actions by intoxicated passengers are foreseeable. The airline has an affirmative duty to protect its passengers from foreseeable harmful acts of third parties caused by conditions that the airline created. This is the best answer because it addresses the airline's proactive responsibility to its invitees for the conditions it created.

A company operated an installation for distributing sand and gravel. The installation was adjacent to a residential area. On the company's grounds, there was a chute with polished metal sides for loading sand and gravel into trucks. The trucks being loaded stopped on the public street below the chute. After closing hours, a plywood screen was placed in the chute and the ladder used for inspection was removed to another section of the installation. For several months, however, a number of children, 8 to 10 years of age, had been playing on the company's property and the adjoining street after closing hours and had discovered the chute could be used as a slide. The company knew of this activity. One evening, as children were playing in the chute, a commuter driving by the chute hit an 8-year-old boy who slid down in front of the automobile. The commuter applied her brakes, but they suddenly failed, and she hit and injured the child. The commuter saw the child in time to have avoided hitting him if her brakes had worked properly. Two days earlier, the commuter had taken her car to a mechanic to have her brakes inspected, and the mechanic had told her that the brakes were in perfect condition. Claims were asserted on behalf of the child by his proper legal representative against the company, the commuter, and the mechanic. With respect to the child's claim against the commuter, the commuter's best defense is that

she used reasonable care in the maintenance of her brakes. The prima facie case for negligence requires a duty, a breach of that duty, causation, and damages. Here, the commuter had a general duty of care: to act as an ordinary, prudent, and reasonable driver. There is no indication that the commuter was negligent in her driving. Instead, it was the state of the car's brakes that caused the injury. Therefore, if the commuter used reasonable care in the maintenance of her brakes, she will have satisfied her duty and not be liable for the child's injuries.

The Federal Automobile Safety Act established certain safety and performance standards for all automobiles manufactured in the United States. The Act creates a five-member "Automobile Commission" to investigate automobile safety, to make recommendations to Congress for new laws, to make further rules establishing safety and performance standards, and to prosecute violations of the act. The chairman is appointed by the President, two members are selected by the President pro tempore of the Senate, and two by the Speaker of the House of Representatives. A minor United States car manufacturer seeks to enjoin enforcement of the Commission's rules. The best argument that the car manufacturer can make is that

the Commission lacks authority to enforce its standards because not all of its members were appointed by the President. Congress has the power to create agencies and to delegate power to those agencies. However, Congress cannot appoint members of a body with administrative or enforcement powers. Since enforcement is an executive power, the fact that some of the commission is appointed by Congress voids that power. Thus, the best argument the car manufacturer has is that the appointments by Congress would prohibit the Commission from being able to enforce its standards.

A landowner owned and occupied Blackacre, which was a tract of land improved with a one-family house. His friend orally offered him $50,000 for Blackacre, the fair market value, and he accepted. Because they were friends, they saw no need for attorneys or written contracts and shook hands on the deal. The friend paid the landowner $5,000 down in cash and agreed to pay the balance of $45,000 at an agreed closing time and place. Before the closing, the friend inherited another home and asked the landowner to return his $5,000. The landowner refused, and, at the time set for the closing, the landowner tendered a good deed to the friend and declared his intention to vacate Blackacre the next day. The landowner demanded that the friend complete the purchase. The friend refused. The fair market value of Blackacre has remained $50,000. In an appropriate action brought by the landowner against the friend for specific performance, if the landowner loses, the most likely reason will be that

the agreement was oral The question asks you to analyze why the court would rule against the landowner. This is essentially a hypothetical, asking you IF the court did something, WHY did they do it. To tackle a question like this, you have to look at all the answer choices and decide which scenario would make the hypothetical situation stated in the call occur. The Statute of Frauds mandates that any contract for the sale of land must be in writing and signed by the party to be charged. In this case, the friend's reasons for refusing to close and the landowner's justifications for mandating performance are irrelevant because the sale contract was never memorialized in a writing. A land sale contract may never be oral, so answer A is correct. Part performance that unequivocally indicates that the parties have contracted for the sale of land may remove the contract out of the Statue of Frauds. What constitutes sufficient part performance varies among jurisdictions. The majority requires at least two of the following: (i) payment (in whole or in part); (ii) possession; and/or (iii) valuable improvements. In this case, there was a partial payment, but not possession or valuable improvements. Therefore, the Statute of Frauds still applies to this case.

A statute in a jurisdiction makes it a crime to sell ammunition to a minor (defined as a person under the age of eighteen). The courts have interpreted this statute as creating a strict liability offense that does not require knowledge of the age of the purchaser and as creating vicarious liability. A minor, who was sixteen years old, but looked four or five years older, entered a store and asked a clerk for a box of .22 caliber shells. The store owner had instructed her employees not to sell ammunition to minors. The clerk asked the minor his age. The minor said he was twenty. The clerk then placed a box of shells on the counter and asked, "Anything else?" The minor said that was all he wanted but then discovered he did not have enough money to pay for the shells, so the clerk put the box back onto the shelf. If the owner of the store is charged with attempting to violate the statute, her best argument would be that

the clerk did not have the mental state needed for attempt. To be convicted of attempt, a person must have the intent that the crime be committed, and take a substantial step toward the commission of the offense. In this case, the clerk did not have the intent to sell ammunition to minors, and the store owner, as his accessory, should be found not guilty of attempting to sell ammunition to minors.

The owner in fee simple of Highacre, an apartment house property, entered into an enforceable written agreement to sell Highacre to a buyer. The agreement provided that a good and marketable title was to be conveyed free and clear of all encumbrances. However, the agreement was silent as to the risk of fire prior to closing, and there is no applicable statute in the state where the land is located. The premises were not insured. The day before the scheduled closing date, Highacre was wholly destroyed by fire. When the buyer refused to close, the seller brought an action for specific performance. If the seller prevails, the most likely reason will be that

the doctrine of equitable conversion applies With regard to real estate purchases, the time period between contracting and closing is governed by the doctrine of equitable conversion. Once the contract is signed, the risk of loss for any damage to the property that: (i) occurs before the closing, and (ii) is not the seller's fault, will fall on the buyer. This is true regardless of whether the seller still physically possesses the land. That is because through the doctrine of equitable conversion, title to the property has been "converted" to the buyer through the operation of equitable principles. Because the buyer bears the risk after contracting, and the facts do not indicate the seller was responsible for the fire damage that occurred, the most likely reason for the seller to prevail will be the application of the doctrine of equitable conversion, so answer D is correct.

A driver negligently drove his car into a pedestrian, breaking her leg. The pedestrian's leg was put in a cast, and she used crutches to get around. While shopping at her local supermarket, the pedestrian non-negligently placed one of her crutches on a banana peel that had been negligently left on the floor by the manager of the supermarket's produce department. The pedestrian's crutch slipped on the peel, and she fell to the floor, breaking her arm. Had the pedestrian stepped on the banana peel at a time when she did not have to use crutches, she would have regained her balance. The pedestrian sued the driver and the supermarket for her injuries. The pedestrian will be able to recover from

the driver, for both of her injuries. The pedestrian's broken arm is proximately connected to the driver's negligent driving. Injuries stemming from being hit by a car include broken bones. The manner in which the arm was broken is not considered, only the injury itself. Courts generally hold that subsequent negligent actions, which may add to the plaintiff's injuries, are a foreseeable consequence of the original action and will not break the chain of causation. The facts clearly state that the pedestrian would not have lost her balance had she not been on crutches as the result of being hit by the driver. Because it is foreseeable that walking on crutches would make the pedestrian less stable on her feet and more vulnerable to subsequent injury from falling, the driver is the proximate (legal) cause of both injuries (although responsibility for the broken arm alone may be apportioned between the driver and the supermarket as concurrent tortfeasors).

Several years ago, a carpenter purchased Goldacre, financing a large part of the purchase price by a loan from a bank that was secured by a mortgage. The carpenter made the installment payments on the mortgage regularly until last year. Then the carpenter persuaded a friend to buy Goldacre, subject to the mortgage to the bank. They expressly agreed that the friend would not assume and agree to pay the carpenter's debt to the bank. The carpenter's mortgage to the bank contained a due-on-sale clause stating, "If Mortgagor transfers his/her interest without the written consent of Mortgagee first obtained, then at Mortgagee's option the entire principal balance of the debt secured by this Mortgage shall become immediately due and payable." However, without seeking the bank's consent, the carpenter conveyed Goldacre to the friend, the deed stating in pertinent part ". . . subject to a mortgage to the bank [giving details and recording data]." The friend took possession of Goldacre and made several mortgage payments, which the bank accepted. Now, however, neither the friend nor the carpenter has made the last three mortgage payments. The bank has brought an appropriate action against the friend for the amount of the delinquent payments. In this action, judgment should be for

the friend, because she did not assume and agree to pay the carpenter's mortgage debt The mortgagor of a property is generally free to transfer title to that property BUT: 1) the mortgagor remains personally liable on the mortgage; and 2) all subsequent grantees take the property subject to the mortgage. Further, subsequent grantees do not become personally liable on the mortgage unless they explicitly assume the mortgage. Therefore, the mortgagor will always be personally liable on the mortgage, all subsequent grantees who assume the mortgage will be personally liable on it, and all subsequent grantees (whether they assume the mortgage or not) can lose the property through foreclosure if the mortgage is not paid. The facts state that the friend did not assume the mortgage, so she has no personal liability to the bank with regard to mortgage payments. Although she voluntarily made such payments for a few months, she was under no obligation to do so. Therefore, answer A is correct. The bank, however, has the right to foreclose on the mortgage if the payments are not made, so the friend stands to lose the property unless either she or the carpenter makes the payments. Further, the carpenter's conveyance without the bank's permission allows the bank, pursuant to the due-on-sale clause, to demand immediate payment of the entire outstanding mortgage balance.

A patron ate a spicy dinner at a restaurant on Sunday night. He enjoyed the food and noticed nothing unusual about the dinner. Later that evening, the patron had an upset stomach. He slept well through the night, went to work the next day, and ate three meals. His stomach discomfort persisted, and by Tuesday morning he was too ill to go to work. Eventually, the patron consulted his doctor, who found that the patron was infected with a bacterium that can be contracted from contaminated food. Food can be contaminated when those who prepare it do not adequately wash their hands. The patron sued the restaurant for damages. He introduced testimony from a health department official that various health code violations had been found at the restaurant both before and after the patron's dinner, but that none of the restaurant's employees had signs of bacterial infection when they were tested one month after the incident. The restaurant's best argument in response to the patron's suit would be that

the patron has failed to establish that the restaurant's food caused his illness. The defense with the best chance of prevailing. Health code violations can only establish duty and breach; they do not establish the restaurant's causal control over the specific instrumentality that caused the actual food poisoning. In addition, res ipsa loquitur does not apply because the patron was unable to show that the restaurant had exclusive control over everything the patron ate within the period leading up to his illness. As a consequence, the restaurant's negligence cannot be inferred by the circumstances.

A man and a woman planned to hold up a bank. They drove to the bank in the man's car. The man entered while the woman remained as lookout in the car. After a few moments, the woman panicked and drove off. The man looked over the various tellers, approached one and whispered nervously, "Just hand over the cash. Don't look around, don't make a false move--or it's your life." The teller looked at the fidgeting man, laughed, flipped him a dollar bill and said, "Go on, beat it." Flustered, the man grabbed the dollar and left. Soon after leaving the scene, the woman was stopped by the police for speeding. Noting her nervous condition, the police asked the woman if they might search the car. She agreed. The search turned up heroin concealed in the lid of the trunk. The prosecution's best argument to sustain the validity of the search of the man's car would be that

the woman had, under the circumstances, sufficient standing and authority to consent to the search. The woman, as the driver and sole person in the vehicle, had standing and the apparent authority to consent to the search of the vehicle, even if she did not own the vehicle. By obtaining the woman's consent to search the vehicle, the police acted reasonably, and the evidence should be admitted.

A homeowner met a man, who was known to him to be a burglar, in a bar. The homeowner told the man that he needed money. He promised to pay the man $500 if the man would go to the homeowner's house the following night and take some silverware. The homeowner explained to the man that, although the silverware was legally his, his wife would object to his selling it. The homeowner pointed out his home, one of a group of similar tract houses. He drew a floor plan of the house that showed the location of the silverware. The homeowner said that his wife usually took several sleeping pills before retiring, and that he would make sure that she took them the next night. He promised to leave a window unlocked. Everything went according to the plan except that the man, deceived by the similarity of the tract houses, went to the wrong house. He found a window unlocked, climbed in and found silver where the homeowner had indicated. He took the silver to the cocktail lounge where the payoff was to take place. At that point, police arrested the two men. If the homeowner was charged with burglary, his best argument for acquittal would be that

there was no intent to commit a felony. Burglary is defined at common law as the breaking and entering of a dwelling of another with the intent to commit a felony therein. Neither the homeowner nor the man had the intent to commit a felony, which should lead to an acquittal for the homeowner. The man's mistake regarding which house belonged to the homeowner negates the intent required for burglary. Therefore, this is also the homeowner's best argument for acquittal.

A particular state has a state employee grievance system that requires any state employee who wishes to file a grievance against the state to submit that grievance for final resolution to a panel of three arbitrators chosen by the parties from a statewide board of 13 arbitrators. In any given case, the grievant and the state alternate in exercising the right of each party to eliminate five members of the board, leaving a panel of three members to decide their case. At the present time, the full board is composed of seven male arbitrators and six female arbitrators. A female state employee filed a sexual harassment grievance against her male supervisor and the state. The state's attorney exercised all of her five strikes to eliminate five of the female arbitrators. At the time she did so, the state's attorney stated that she struck the five female arbitrators solely because she believed women, as a group, would necessarily be biased in favor of another woman who was claiming sexual harassment. Counsel for the state employee eliminated four males and one female arbitrator, all solely on the grounds of specific bias or conflicts of interest. As a result, the panel was all male. When the panel ruled against the state employee on the merits of her case, she filed an action in an appropriate state court, challenging the panel selection process as a gender-based denial of equal protection of the laws. In this case, the court should hold that the panel selection process is

unconstitutional, because the gender classification used by the state's attorney in this case does not satisfy the requirements of intermediate scrutiny The state's attorney in this case intentionally excluded arbitrators based on gender. The Supreme Court has held that peremptory challenges based solely on gender are unconstitutional as a violation of the Equal Protection Clause because they reinforce negative stereotypes against women without furthering the governmental interest of a fair trial. See J.E.B. v. Alabama, 511 U.S. 127 (1994). Similarly, the state's attorney in this question is making a gender-based classification, so the state would need to show that the classification is substantially related to an important government objective. Because the state's attorney's argument was that women would simply be biased toward a female claimant, the attorney failed to demonstrate how the exclusion of female arbitrators would be substantially related to the need for a fair arbitration.

A particular state has a state employee grievance system that requires any state employee who wishes to file a grievance against the state to submit that grievance for final resolution to a panel of three arbitrators chosen by the parties from a statewide board of 13 arbitrators. In any given case, the grievant and the state alternate in exercising the right of each party to eliminate five members of the board, leaving a panel of three members to decide their case. At the present time, the full board is composed of seven male arbitrators and six female arbitrators. A female state employee filed a sexual harassment grievance against her male supervisor and the state. The state's attorney exercised all of her five strikes to eliminate five of the female arbitrators. At the time she did so, the state's attorney stated that she struck the five female arbitrators solely because she believed women, as a group, would necessarily be biased in favor of another woman who was claiming sexual harassment. Counsel for the state employee eliminated four males and one female arbitrator, all solely on the grounds of specific bias or conflicts of interest. As a result, the panel was all male. When the panel ruled against the state employee on the merits of her case, she filed an action in an appropriate state court, challenging the panel selection process as a gender-based denial of equal protection of the laws. In this case, the court should hold that the panel selection process is

unconstitutional, because the gender classification used by the state's attorney in this case does not satisfy the requirements of intermediate scrutiny. The state's attorney in this case intentionally excluded arbitrators based on gender. The Supreme Court has held that peremptory challenges based solely on gender are unconstitutional as a violation of the Equal Protection Clause because they reinforce negative stereotypes against women without furthering the governmental interest of a fair trial. See J.E.B. v. Alabama, 511 U.S. 127 (1994). Similarly, the state's attorney in this question is making a gender-based classification, so the state would need to show that the classification is substantially related to an important government objective. Because the state's attorney's argument was that women would simply be biased toward a female claimant, the attorney failed to demonstrate how the exclusion of female arbitrators would be substantially related to the need for a fair arbitration.


Conjuntos de estudio relacionados

Challenge 1.3 Conflict Resolution

View Set

Other Words for Home by Justine Warga

View Set

human bio exam 5, lab and mastering

View Set

Chapter 6 - Markets and Social Security

View Set

Chapter 11: International Strategy and Organization

View Set

Microscopy- Use and Function in Anatomy

View Set